i-c8eb72d578ba0c74248a904a25b283e4-Dyson.cleaner.dc07.arp-thumb-100x182.jpg

Ich lese gerade ein sehr gutes Buch über schwarze Löcher (Rezension folgt). Dadurch bin ich wieder auf eines der hauptsächlichen Mißverständnisse über schwarze Löcher gestoßen die man sehr oft hört (zum Beispiel in den Diskussionen über den LHC).

Viele Leute glauben anscheinend, ein schwarzes Loch wäre sowas ähnliches wie ein kosmischer Staubsauger, das alles was in seine Nähe kommt ansaugt und verschwinden lässt…

Das ist nicht wirklich korrekt. Ein schwarzes Loch ist erstmal ein Objekt mit einer bestimmten Masse (genaugenommen hat ein schwarzes Loch außer der Masse auch kaum mehr weitere Eigenschaften) und verhält sich auch so wie jedes andere massebehaftete Objekt.

Das bedeutet, es hält sich so wie alle anderen Objekte – Sterne, Planeten, Asteroiden, Äpfel – an die Gravitationsgesetze (Wie gesagt: näherungsweise. Bei näherer Betrachtung wird die Sache viel komplizierter).

Würde es plötzlich “plopp” machen und die Sonne würde sich spontan in ein gleich schweres schwarzes Loch verwandeln, dann würde wir auf der Erde nicht viel davon merken. Ok, es würde dunkel werden und sehr kalt – aber die Erde würde sich genauso um das schwarze Loch bewegen wie vorher um die Sonne. Die Masse ist ja gleich geblieben und daher auch die Gravitationskräfte, die zwischen Erde und schwarzem Loch wirken.

Die Vorstellung, das schwarze Loch würde nun, nur weil es ein schwarzes Loch ist, plötzlich anfangen zu “saugen” und die Erde wäre in Gefahr ist falsch. Physikalisch würde das ja bedeuten, dass auf die Erde auf einmal eine größere Gravitationskraft wirkt – und das geht nur, wenn sich die Masse des schwarzen Lochs erhöht hätte.

Das, was ein schwarzes Loch so außergewöhnlich macht, ist eine andere Eigenschaft: seine Dichte! Um aus der Sonne ein schwarzes Loch zu machen müsste man ihre gesamte Masse innerhalb einer etwa drei Kilometer großen Sphäre konzentrieren (übrigens wird aus der Sonne auf natürlichen Weg nie ein schwarzes Loch werden – das passiert nur mit größeren Sternen).

i-b7b0b5d8943e63e63282e62331e1a20f-Spacetime_curvature-thumb-500x220.png

Massen krümmen die Raumzeit (Bild: Johnstone, GFDL 1.2)

In einem schwarzen Loch ist also Masse auf einem extrem kleinen Raum konzentriert. Nach der allgemeinen Relativitätstheorie wird die Raumzeit durch Massen gekrümmt. Ein schwarzes Loch mit Sonnenmasse krümmt den Raum genauso wie die Sonne selbst und wenn man ausreichend weit entfernt ist – so wie die Erde – dann merkt man keinen Unterschied. Die “Mulde” in der Raumzeit (es ist einfacher, wenn man hier nur in zwei Dimensionen denkt) ist bei der Sonne allerdings wesentlich “breiter” und “flacher” als beim schwarzen Loch.

Erst die extreme Dichte – und nicht etwa eine besonders starke “saugende” Gravitationskraft – eines schwarzen Loches führt dazu, dass die Raumzeit so stark gekrümmt wird, dass etwas, das einmal eine bestimmte Grenze überschritten hat (den Ereignishorizont) nicht mehr zurück kann – nicht einmal Licht.

i-0231cc03291ba78d601134e599d9241b-schwarzesloch-thumb-500x268.jpg

Ein schwarzes Loch krümmt den Raum besonders stark (Bild: AllenMC, Creative Commons 3.0 Unported)

Deswegen sind auch potentielle kleine schwarze Löcher die unter ganz bestimmten Umständen (unser Universum müsste dafür mehr als nur 3 Raumdimensionen haben) am Teilchenbeschleuniger LHC entstehen könnten keine Gefahr. Menschen, die an die “Staubsauger-Löcher” glauben stellen sich vor, dass es für den Weltuntergang reicht, so ein kleines schwarzes Loch entstehen zu lassen. Danach würde es sofort die ganze Erde “einsaugen”.

Aber das ist, so wie ich oben erklärt habe, natürlich falsch. So ein kleines schwarzes Loch wäre extremst klein mit einer enorm geringen Masse und einer dem entsprechenden geringer Gravitationskraft. Das Loch wäre so winzig, dass aus seiner Sicht selbst feste Materie aus sehr viel “Nichts” mit ein paar Elementarteilchen drin bestehen würde. Elementarteilchen, die es wegen seiner geringer Masse nicht mal wirklich gravitativ beeinflussen kann. Es müsste also warten, bis ihm zufällig mal ein verirrtes Elektron nahe kommt und mit ihm kollidiert. Dann hätte es eine geringfügig größere Masse – und das Spiel geht von vorne los. Bis das schwarze Loch auf diese Art massiv genug wäre, um der Erde tatsächlich Schaden zuzufügen, hätte die sterbende Sonne die Erde längst vernichtet.

Kommentare (146)

  1. #1 schlappohr
    12. Januar 2010

    Ich habe mal irgendwo eine sehr interessante Analogie dazu gelesen. Jemand hat die
    Wärmenenergie einer Badewanne (1.5m^3) mit 35°C warmem Wasser berechnet und überlegt, was man mit dieser Energiemenge anstellen könnte, wenn man sie auf ein kleineres Volumen konzentiert (die Zahlen stammen nur aus der Erinnerung und sind wahrscheinlich falsch):

    – Bei einem Volumen von 0.5m^3 würde das Wasser verdampfen
    – Bei 10cm^3 würde Aluminium schmelzen
    – Bei 0,000irgendwas mm^3 könnte man eine Kernfusion einleiten

    Und das alles mit der Wärmeenergie einer vollen Badewanne. Es kommt eben nur auf die Energiedichte an.

  2. #2 Snake
    12. Januar 2010

    Also ist ein schwarzes Loch doch wie jede andere größere Masse eben doch ein kosmischer Staubsauger, der alles einfängt, was nicht genug Energie mitbringt. 😉

    Nur eine Kleinigkeit noch zu deinem letzten Absatz: Wie würde so ein Mini-Loch vorbeifliegende Elektronen einfangen? Ich war bisher der Meinung, dass in dieser Größenordnung vor allem die elektromagnetischen Wechselwirkungen dominieren und Elektronen aufgrund ihrer geringen Masse sowieso kaum über Gravitation wechselwirken.

    Also entweder müsste das Elektron dem schwarzen Loch zufällig wirklich sehr nahe kommen, bis dieses von der lokal sehr starken Gravitation des Lochs (in der Zeichnung oben als Gradient dargestellt?) eingefangen wird, oder das Loch hat eine (positive) Ladung und wirkt elektrisch. Oder habe ich da was falsch verstanden?

    Falls das Elektron dennoch eingefangen wird, müsste sich doch im Loch nach und nach ein Überschuss an negativen Ladungsträgern ansammeln und irgendwann die elektrische Wechselwirkung der Gravitationskraft entgegenwirken, so dass keine weiteren Elektronen mehr eingfangen werden können und das Wachstum zum erliegen kommt, oder?

  3. #3 beme
    12. Januar 2010

    Sind Sie sich da sicher?
    Ich stell mir gerade vor Richtung Erdmittelpunkt gesaugt äh gezogen zu werden.

  4. #4 Christian W
    12. Januar 2010

    Wieder einmal ein schönes Beispiel dafür, warum Fachleute und Laien nicht miteinander kommunizieren können. Wenn Laien Fachleute etwas fragen, wollen sie einfache Antworten hören. Gefällt ihnen die Antwort dann nicht, kommt unausweichlich die “Ja aber was ist mit…?”-Frage und die Probleme beginnen. Hier eben u.a.:

    Menschen, die an die “Staubsauger-Löcher” stellen sich vor, dass es für den Weltuntergang reicht, so ein kleines schwarzes Loch entstehen zu lassen. Danach würde es sofort die ganze Erde “einsaugen”.

    Tatsächlich glauben Laien solche Dinge und setzen sie bei ihren Fragen als evident voraus. Widerspricht nun die Fachmann-Antwort ihren vorausgesetzten Annahmen, werden nicht etwa die eigenen als Wissen betrachteten Annahmen in Frage gestellt (weil die ja aus Star Trek, P.M. und WWW abgesichert sind), sondern die Fachmann-Antwort. Natürlich lügt der Fachmann, interessant ist nur noch, warum.
    Andersherum kommt der Fachmann nicht umhin festzustellen, dass sich die kognitive Dissonanz, die sich beim Laien durch den Konflikt zwischen Fachmann-Antwort und seinen eigenen Vorwegannahmen ergeben hat, nur dadurch auflösen ließe, dass er (Fachmann) die ganzen falschen Annahmen richtig stellt und dem Laien Berge von Erklärungen lieftert – zu allem was auch nur ansatzweise mit seiner Frage zu tun hat. Der wiederum ist daran natürlich überhaupt nicht interessiert und fühlt sich gegängelt, reagiert sogar wütend: “Kannst du nichtmal eine einfache Frage beantworten, ohne vom Hundertsten in’s Tausendste zu kommen und uns mit deinem Nerd-Wissen zu langweilen?”. Woraufhin dem Fachmann nichts weiter bleibt als zu erklären “Nunja, entweder du machst dich mit den Grundlagen vertraut oder du wirst es nie verstehen. Entweder du vertraust meiner Expertise oder du legst dir selbst eine zu.”, was dem Laien wiederum weitere Nahrung für seine Vermutungen und Spekulationen liefert.

    Und alles nur, weil der Laie ein Laie ist und der Fachmann ein Fachmann. Wenn es nicht so dringend nötig wäre, Sachverstand zu vermitteln, bräuchte man mit Kommunikation zwischen Laien und Fachleuten gar nicht erst anfangen.
    Das ist der Grund für meinen wahnsinnigen Respekt vor dir und deiner Geduld, Florian. Du versuchst es immer wieder und immer neu. Das könnte ich nicht. Ich hätte schon längst aufgegeben und würde mir die Zeit mit fiesen Zynismen vertreiben.

  5. #5 Florian Freistetter
    12. Januar 2010

    @beme: Was genau meinst du mit deiner Vorstellung?

    @Snake: Hmm -also wie das mit der Ladung von schwarzen Löchern aussieht kann ich nicht sagen. Dazu bin ich zu wenig Experte auf diesem Gebiet. Ich schau mal, ob ich nach der Lektüre des Buches über schwarze Löcher schlauer bin…

  6. #6 Oliver Debus
    12. Januar 2010

    “Hmm -also wie das mit der Ladung von schwarzen Löchern aussieht kann ich nicht sagen. Dazu bin ich zu wenig Experte auf diesem Gebiet. Ich schau mal, ob ich nach der Lektüre des Buches über schwarze Löcher schlauer bin…”

    Ich will das nur mal am Rande erwähnen: meine Hochachtung für dieses ehrliche Eingeständnis von Florian, dass er auch nicht alles weiss.

  7. #7 ssssssssssssss
    12. Januar 2010

    Der Grund
    “dass etwas, das einmal eine bestimmte Grenze überschritten hat (den Ereignishorizont) nicht mehr zurück kann – nicht einmal Licht. ”
    schreiben Sie, ist die extreme Dichte der Schwarzen Löcher.
    Also große Masse auf auf kleinem Raum.

    Würde man nun plopp machen und aus dem Schwarzen Loch ein “normalen” Himmelskörper machen, z. B. einen ganz ganz großen Stern, also große Masse auf großem Raum, würden Dinge und Licht dann wieder zurück können?

    Ist der Trichter oder die Mulde in der Raumzeit, der Grund für das Nichtzurückkönnen, da die Mulde bei großer Masse auf kleinem Raum tiefer ist als bei großer Masse auf großem Raum?

    Ein schöner Artikel für mich als astromischen und astrologischen Laien.

  8. #8 Marek
    12. Januar 2010

    Hallo Florian,

    für mich stellt sich die Frage ähnlich wie ssssssssssss (hoffe, ich habe den Namen richtig geschrieben). Warum wird bei gleicher Masse einmal das Licht zurückgehalten und einmal nicht?

  9. #9 Bullet
    12. Januar 2010

    Ja, das klingt gut. Im Bezug auf die Sonne als schwarzes Loch sag ich, wenn ich gefragt werde, immer:
    “Auf der Sonnenoberfläche herrscht eine Gravitation von knapp 28 g. Das ist schon recht viel. Wenn man aber in die Sonne eintaucht, ist, je tiefer man kommt, immer mehr Sonnenmasse über einem. Die Anziehungskraft wird also wieder geringer. Wenn man dann die Sonne aber verdichtet = deren Volumen schrumpft, kommt man immer näher an das Zentrum ran, ohne etwas Sonnenmasse über sich zu haben. Und da ja die Kraft gleich M1*M2/r² ist und r (der Abstand) immer kleiner wird, kann der Gesamtterm immer größer werden. Aber für alles, was weiter als 10 Mio. km von der Sonne entfernt ist, isses egal, ob da eine Leuchtsonne oder ein schwarzes Loch im Raum hängt.”

  10. #10 Bullet
    12. Januar 2010

    Oh, da kamen ja so viele Antworten zwischendurch. Sorry. Das hier sollte der zweite Kommentar in der Reihe sein. 🙂

    Aber:
    @Snake:

    Nur eine Kleinigkeit noch zu deinem letzten Absatz: Wie würde so ein Mini-Loch vorbeifliegende Elektronen einfangen? Ich war bisher der Meinung, dass in dieser Größenordnung vor allem die elektromagnetischen Wechselwirkungen dominieren und Elektronen aufgrund ihrer geringen Masse sowieso kaum über Gravitation wechselwirken.

    Genau das isses ja, weswegen dieser LHC-macht-schwarzes-Loch-und-wir-werden-alle-sterben-Dreck Mumpitz ist.
    Es kann schwarze Löcher mit Ladung geben (sagt Wikipedia). Aber ein ungeladenes SW kann nur über Gravitation wechselwirken. Elektronen haben Masse, also sind auch sie der Gravitation ausgeliefert. Jedoch ist die elektromagnetische Kraft um den Faktor …ähm… 10^42 (??) größer. Das heißt: effektiv werden Elektronen viel viel viel viel viel viel eher vom Elektromagnetismus beeinflußt als durch Gravitation. Genau deshalb würde es ja so schweinelange dauern, bis der unglaubliche Zufall diesem Mini-SW mal ein Elektron vor die Nase legt, das NICHT gerade mit dem 10^38-fachen der Kraft des SW irgendwohingezogen wird.
    Und deswegen ist diese Weltuntergangsgeschichte im Zusammenhang mit LHC Mumpitz.

  11. #11 Arnd
    12. Januar 2010

    So ein Mini- schwarzes Loch würde doch aufgrund der Hawking-Strahlung sofort “verdampfen”, oder bin ich da falsch informiert?

  12. #12 Jörg
    12. Januar 2010

    Ich wollte mir auch schon immer nochmal klarmachen, was bedeutet dass ein Schwarzes Loch eine SIngularität in der Zeit, nicht im Raum ist (schreibt Bojowald). Mir wird immer ein bißchen wuselig im Kopf, wenn ich darüber nachdenke…
    Zu der Ladungssache würde ich dann auch sagen, dass es keine elektromagnetische Interaktion mehr geben kann – schließlich braucht es dazu Photonenaustausch, und Photonen kommen nicht raus. Innerhalb des Ereignishorizont vergeht aber die Zeit immer langsamer (von draußen betrachtet), je näher man dem Mittelpunkt kommt. Also sollte sich da die Ladung nicht gegen die Gravitation aufbauen können?
    Im übrigen ist das ja sowieso wieder eine Frage für die man eigentlich Quantengravitation bräuchte…

  13. #13 Bullet
    12. Januar 2010

    @sssssssssss und Marek:

    Wenn wir bei dem Trichter sind, stellt euch folgendes vor:
    Ihr habt ein schwimmfähiges Auto mit 100 PS. Vor euch liegt ein Trichter. (Wie der im Bild von Florians Beitrag.)
    Es gilt:
    – Das Volumen des Trichters ist die Masse der Sonne.
    – Die Steilheit des Trichters an jeder beliebigen Stelle ist die Anziehungskraft, die ihr spürt.
    – Der kleine See im Trichter (den ich jetzt mal grad dazuerfunden hab) ist die sichtbare Sonnenscheibe.
    Es ist klar, daß der Trichter in Richtung auf das Zentrum immer steiler wird. Aber irgendwann kommt ihr an das Ufer des Sees gefahren und schwimmt. Herzlichen Glückwunsch, ihr seid auf der Sonne gelandet. Wenn euer Auto mit 100 PS auf dem Rückweg den Trichter wieder erklimmen kann, dann ist wieder alles gut.
    Jetzt lasse ich Wasser ab. Der Trichter ist jetzt VIEL steiler, die Scheibe der Sonne VIEL kleiner. Wenn ihr jetzt am Wasser ankommt, habt ihr eine ziemliche Sturzfahrt hinter euch. Euer Auto kommt da nicht mehr raus. Wenn euer Auto ein Lichtstrahl ist, habt ihr eben erfahren, was ein schwarzes Loch ist. Dummerweise ist es nicht wirklich einfach, sich vorzustellen, was man tun muß, um all die Materie des schwarzen Loches da wieder auseinanderzuwürgen. Denn alles, was man als Hebel ansetzt, ist ja dann auch im Loch. Aber das einzige, was man tun kann, ist die Materie wieder zu zerstreuen.

    Was allerdings mit dem Auto passiert, weiß ich nicht. Hawking (?) sagt, alles, was in ein SW fällt, ist hinterher nur noch Strahlung. Wenn sich das SW also irgendwann auflöst (da soll es einen Mechanismus geben), wird eine Menge Licht frei.

  14. #14 Snake
    12. Januar 2010

    @marek:
    Die Stärke der Gravitationskraft nimmt mit dem Quadrat der Entfernung zur Masse ab. D.h. ich werde von einem Stückchen Masse in Australien deutlich weniger angezogen als vom gleichen Stückchen Masse direkt unter meinem Haus. Die auf uns wirkende Gravitationskraft der Erde ist also wegen der Ausdehnung der Erde, die im Vergleich zu unserem Abstand zur Erde groß ist, kleiner als wenn wir die Erde als Punktmasse direkt unter unseren Füßen hätten.

    Deshalb kann ein Photon der Sonne mit ihrer Ausdehnung als Stern entkommen, der Sonne als konzentrierte Masse (als schwarzes Loch) aber nicht.

  15. #15 Christian W
    12. Januar 2010

    Würde man nun plopp machen und aus dem Schwarzen Loch ein “normalen” Himmelskörper machen, z. B. einen ganz ganz großen Stern, also große Masse auf großem Raum, würden Dinge und Licht dann wieder zurück können?

    Ich nutze die Frage einfach mal als Probe, ob ich Florian richtig verstanden habe:

    Wenn man dein “plopp” macht, hieße das ja, aus dem räumlich(!) kleinen Loch wieder einen räumlich viel größeren Stern zu machen. Der Unterschied zwischen schwarzen Löchern und Sternen ist doch gerade, dass sie (Löcher) viel, viel, viel dichter sind, aber dieselbe Masse und damit dieselbe Gravitationskraft haben.
    Das bedeutet doch, dass “Dinge und Licht” sehr viel näher an das schwarze Loch “herankommen können” als an einen Stern mit derselben Masse. Somit würde sich rein räumlich dein Versuch nicht durchführen lassen. Der event horizon eines schwarzen Lochs liegt offenbar innerhalb eines “nicht-schwarzen-Lochs” derselben Masse/Gravitationskraft. Das weiß ich zwar nicht, es muss aber so sein, denn sonst hätten ja auch andere Objekte als schwarze Löcher einen event horizon, ab dem alles inklusive Licht “gefangen” wird.

    Oder – um beim “Trichter” zu bleiben – die Trichter des Lochs und des Sterns ist identisch, aber beim Stern gibt es eine physische Annäherungsgrenze, die nicht (auch nicht vom Licht) überschritten werden kann und somit kann die Steigung nicht erreicht werden, von der aus es kein Zurück mehr gibt. Das schwarze Loch hat “sich” hinter diese physische Grenze “zurückgezogen” und so kann die erwähnte Steigung erreicht werden.

    Am I making any sense? Florian?

  16. #16 Florian Freistetter
    12. Januar 2010

    @sssssss, marek: Über das, was mit dem Zeug passiert, das in ein SL fällt, wenn es sich irgendwann mal aufgelöst hat, haben viele Leute nachgedacht. Da gibts ja das Problem mit der Entropie: wenn man etwas mit hoher Entropie ins SL schmeisst, is es quasi weg und die Entropie im Universum sinkt (was sie nicht sollte). Das Problem hat irgendeinen Namen (Ich glaub “Paradoxon” kommt drin vor) und auch eine Lösung – aber mir fällts grad nicht ein 🙁 Vielleicht kann wer anderer aushelfen?

  17. #17 Jörg
    12. Januar 2010

    @Florian: Einfach Information Paradoxon, oder?
    https://en.wikipedia.org/wiki/Black_hole_information_paradox

  18. #18 Bullet
    12. Januar 2010

    Christian W.:
    ich würde sagen, so isses.

  19. #19 Oliver Debus
    12. Januar 2010

    Ich hatte mal einen Vortrag zu dem Thema gemacht. Da hatte ich auch bei Stephen Hawking gelesen, dass Schwarze Löcher Ernergie (Masse) verlieren und sich auflösen können. Das hatte etwas damit zu tun, dass ein Teilchen/Antiteilchen Paar entsteht und eines der beiden ins Loch fällt und das andere entweichen kann und somit dem SL Energie entzieht. Das müßte ich aber noch mal nachlesen, vor allem in welchem Buch von Hawkings das steht.

  20. #20 Marek
    12. Januar 2010

    @Bullet, Snake

    Besten Dank, das hilft schon mal weiter.

    @Florian

    Mir ging es gar nicht darum, was mit den Dingen passiert, die in ein SL “hineinfallen”. Ich wurde nur stutzig bei deiner Aussage, dass, wenn die Sonne mit einem *knülz* zu einem SL werden würde, es auf der Erde lediglich dunkel und etwas kühler werden und die Erde weiter ihre Bahn ziehen würde. Selbe Masse = selbe Gravitation, leuchtet ein.

    Aber bei einem SL muss ja etwas anders sein, da das Licht nicht mehr entkommen kann. Nur an der Gravitation kann es ja nicht liegen.

    Was passiert z.B. mit Merkur? Wird der dann stärker “angesaugt”?

  21. #21 fatmike182
    12. Januar 2010

    Gute Anti-Analogie, das mit dem Staubsauger (also Nicht-Staubsauger)!
    Ich bin gestern zufällig über einen kleinen Artikel gestolpert – da wollen Leute an einer indischen Uni “Schwarze Löcher” für Licht sichtbarer Wellenlänge basteln. Für Mikrowellen soll es schon funktionieren.
    https://www.newscientist.com/article/dn17980

  22. #22 Bullet
    12. Januar 2010

    Doch. Es liegt NUR an der Gravitation. Merkur kann nicht stärker angesaugt werden. Woher soll die zusätzliche Kraft denn kommen? Es gerht nur darum, daß ein SL viel kleiner ist – bei gleicher Masse, und du daher näher rankommst..

  23. #23 Florian Freistetter
    12. Januar 2010

    @Marek: “Was passiert z.B. mit Merkur? Wird der dann stärker “angesaugt”?”

    Eben nicht! Das was besonders am SL ist, passiert erst, wenn man nahe dran ist; in der Nähe des Ereignishorizonts. Die Planeten spüren nur die Gravitation – und die ist gleich; egal ob sie von einem SL oder der Sonne kommt. Das “anders sein” liegt an der höheren Dichte des SL.

    @Jörg: Danke, ja genau das wars 😉

    @Christian W.: Klingt gut!

  24. #24 Oliver Debus
    12. Januar 2010

    @ Marek
    “Was passiert z.B. mit Merkur? Wird der dann stärker “angesaugt”?”
    Ich will Florian nicht vorgreifen, aber nein. Merkur wird nicht stärker angesaugt. vorrausgesetzt die Masse der Sonne bleibt gleich, ändert sich an ihrer Gravitationskraft nichts. Nur wenn ihre Masse zunimmt nimmt auch ihre Gravitationskraft zu. An ihrer neuen Oberfläche ist die Gravitation zwar sehr stark, aber die Gravitationskraft schwächt sich mit dem Quadrat des Abstandes ab.

  25. #25 Christian W
    12. Januar 2010

    @Bullet, Florian
    Oh gut, das freut mich. Normal brauche ich länger, gerade wenn es um Singularitäten und andere Sachen geht, wo Einstein, Planck, Feynman und Hawking nur “educated guessen” und herumrechnen können… 😉
    Danke für’s Korrekturlesen.

  26. #26 Christian W
    12. Januar 2010

    Marek, nimm’ einfach einmal den “leeren” Trichter von oben. Stell’ dir vor, der “3. Ring von innen” ist der event horizon und das SL beginnt am innersten sichtbaren Ring. Der zugehörige Stern dagegen beginnt schon am zweiten sichtbaren Ring von außen.

    Wie soll irgendwas bis zum event horizon kommen? Und was ändert sich für Objekte am äußersten sichtbaren Ring?

  27. #27 Moss
    12. Januar 2010

    @fatmike182: diese Tarnkappengeschichten gibt’s, das waren allerdings Forscher an der Duke University 2007. Im optischen Bereich haben da die Leute von der Uni Stuttgart 2008 was vorgestellt (ganz unten auf der Seite).

    Mit Black Holes oder überhaupt Gravitationsphysik hat das jetzt aber garnix zu tun! 😉

  28. #28 Marek
    12. Januar 2010

    Also kann man einem SL genauso nahe kommen wie einem Stern mit gleicher Masse; die Konsequenzen sind dieselben? Erst ab dem Ereignishorizont: *pfump*. Tut mir leid, ich bin etwas verwirrt … 🙂

  29. #29 Christian W
    12. Januar 2010

    Also kann man einem SL genauso nahe kommen wie einem Stern mit gleicher Masse; die Konsequenzen sind dieselben? Erst ab dem Ereignishorizont: *pfump*. Tut mir leid, ich bin etwas verwirrt … 🙂

    Kein Problem, du hast es gleich: Ja, einem SL kannst du gefahrlos genauso nah kommen wie einem masseidentischen Stern (von beider Mittelpunkt aus, natürlich nicht vom äußeren Rand 😉 ), ohne dass irgendetwas (abgesehen von der Beleuchtung und Temperatur) anders wäre. Erst ab dem Ereignishorizont wird’s gefährlich.

    Mein Gedankenexperiment von oben war vielleicht noch nicht perfekt, darum eine weitere Variante. Stell dir ein SL genau so wie oben abgebildet vor, also mit tatsächlichem Loch (im Raum) in der Mitte. Gehst du “über die Kante”, fällst du hinein und kannst an den glitschigen Wänden nicht mehr hochkrabbeln. Wenn statt des Lochs aber ein Stern mit gleicher Masse an derselben Stelle wäre, würde der das Loch verstopfen und du könntest gar nicht hineinfallen, egal wie nah du an den Stern herangehst. 🙂

  30. #30 Bullet
    12. Januar 2010

    Schon komisch … hier schreiben Leute, die anderen Leuten etwas erklären, und hinterher verstehen diese anderen Leute, was gemeint ist. Fragen werden beantwortet und führen zu Informationszuwachs. Warum funktioniert das nicht, wenn angebliche Doktoren Halb- und Zweifünftelfakten zu einer verqueren Melange zusammenmischen und sich daraus …. ach, egal. Weitermachen. 🙂

  31. #31 Marek
    12. Januar 2010

    @Christian W

    Das mit den Gedankenexperimenten ist immer so eine Sache. Ich grüble lieber noch etwas über den Trichter nach. Das Beispiel mit dem (verstopften) Loch finde ich jetzt nicht so geschickt, tschuldige. 🙂

  32. #32 Christian W
    12. Januar 2010

    Warum, was ist an dem verstopften Loch unglücklich? Keine Angst, du kannst mir ruhig ehrlich und direkt sagen, wenn ich Unsinn schreibe. Dabei kann ich nur lernen und vielleicht eine viel bessere Erklärung finden.

    Was gibt dir am Trichter Grund zum Grübeln? Die Übersetzung des zweidimensionalen Gedankenbildes in gekrümmten Raum? Da kann ich dir auch nicht weiterhelfen. Ich habe mal versucht, das zu verstehen – hat aber nie geklappt. Ich vertraue einfach mal den Fachleuten, dass das eine gute Approximation der Realität ist.

  33. #33 Jörg
    12. Januar 2010

    wenn angebliche Doktoren Halb- und Zweifünftelfakten zu einer verqueren Melange zusammenmischen und sich daraus

    Haha, das klingt aber exakt wie der Prozess wenn ich etwas über Schwarze Löcher, Quanten- oder Festkörperphysik schreibe 😀

  34. #34 Bullet
    12. Januar 2010

    Ehrlich gesagt, find ich das verstopfte Loch auch eine gute Analogie. 🙂

    Jörg:
    schreibst du häufiger über Quantenphysik? *g*

  35. #35 Marek
    12. Januar 2010

    @Christian W

    Was gibt dir am Trichter Grund zum Grübeln? Die Übersetzung des zweidimensionalen Gedankenbildes in gekrümmten Raum?

    Ja, auch das ist nicht leicht zu verstehen. Ist aber nicht der Punkt, um den es mir geht.

    Ich bin immer davon ausgegangen – wie so viele vermutlich -, dass ein SL eine extrem hohe Anziehungskraft hat. Gut, hat es, aber eben nicht mehr als ein Stern mit gleicher Masse.
    Nur ab einem bestimmten Punkt – dem Ereignishorizont – sieht es anders aus. Wegen der Dichte und der Art der Raumzeitkrümmung. Und das versuche ich mir nun vorzustellen. Trichter, Löcher – vielleicht macht es heute noch *schnipp*. 🙂

  36. #36 Bullet
    12. Januar 2010

    Marek: denk dir ein Lagerfeuer mit einem Zaun drum. Normale Lagerfeuer haben sowas aus Sicherheitsgründen. Jeder, der wissen möchte, wie warm so ein Lagerfeuer ist, kann an den Zaun treten und mit einem Thermometer messen.
    Jetzt nehm ich den Zaun weg … plötzlich kannst du dein Thermometer direkt in die Flamme halten und wirst feststellen, daß du andere, deutlich höhere Temperaturen bekommst. In der Entfernung, in der der Zaun früher war, ist keine Veränderung zu sehen. Aber du hast jetzt die Möglichkeit, näher heranzugehen. (Schon wieder eine Umsetzung mit “nah”. Dammich!) Und dann bekommst du auch leicht andere Meßergebnisse. Solche allerdings, die du prinzipiell schon kennst: eingefangene Wärme ~> 1/r². Das war vor dem Zaun auch schon so. Du hast jetzt nur höhere Zahlen. Mehr nicht.

  37. #37 Christian W
    12. Januar 2010

    Nur ab einem bestimmten Punkt – dem Ereignishorizont – sieht es anders aus. Wegen der Dichte und der Art der Raumzeitkrümmung.

    Nein, nicht wegen der “Art der Raumzeitkrümmung” – einzig und allein wegen der Dichte. Du hast bei einem SL eben nicht eine gewaltige Masse x auf einem gewaltigen Raum y (etwa Beteigeuze) verteilt, sondern dieselbe Masse x auf y/1.000.000.000 verteilt. Darum haben SL einen “frei zugänglichen” Ereignishorizont und Sterne (wie andere Objekte) nicht. Weil es bei SL Orte gibt, die so nah am Mittelpunkt des Objekts sind und trotzdem noch nicht im Objekt.

    Weiter oben hat Bullet vorhin schon eine weniger plastische, dafür aber vielleicht besser nachvollziehbare Erklärung dieses Unterschieds zwischen SL und Sternen verfasst:

    “Auf der Sonnenoberfläche herrscht eine Gravitation von knapp 28 g. Das ist schon recht viel. Wenn man aber in die Sonne eintaucht, ist, je tiefer man kommt, immer mehr Sonnenmasse über einem. Die Anziehungskraft wird also wieder geringer. Wenn man dann die Sonne aber verdichtet = deren Volumen schrumpft, kommt man immer näher an das Zentrum ran, ohne etwas Sonnenmasse über sich zu haben. Und da ja die Kraft gleich M1*M2/r² ist und r (der Abstand) immer kleiner wird, kann der Gesamtterm immer größer werden. Aber für alles, was weiter als 10 Mio. km von der Sonne entfernt ist, isses egal, ob da eine Leuchtsonne oder ein schwarzes Loch im Raum hängt.”

  38. #38 stefan
    12. Januar 2010

    Ich war immer schon der meinung das der beliebte button-spruch “black holes suck”, in seiner absolutheit seiner aussage, einfach nur falsch ist. 😉

    die “dunkelheit” lässt sich nun auch leicht berechnen: auf https://xaonon.dyndns.org/hawking/ gibt es einen (nicht relativistischen, sonst wärs sehr aufwendig ;-)) online-Rechner für die Hawking-Strahlung.

    demnach gäbe es folgende kennzahlen eines Schwarzen Loches mit einer Sonnenmasse:
    Radius 2954.855 meters
    Surface area 109.7191 square kilometers
    Surface gravity 1.520811e+13 meters/second^2
    Surface tides 1.049660e+9 Earth gravities / meter
    Entropy 4.561405e+76 (dimensionless)
    Temperature -273.1500 celsius
    Luminosity 8.998363e-29 watts
    Lifetime 2.099496e+67 years

    Entropy – damit ist wohl die fuer BH modifizierte Informationstheorie-Entropie gemeint, denn die thermodynamische ist ja nicht dimensionslos.

    Wärend ein Schwarzes Loch mit knapp einer Tonne schon so viel strahlt wie die Sonne (~10^26 Watt), aber gerade mal 7.342739e-8 seconds lebenserwartung hat.

    Diese Rechnungen zeigen auch sehr schön wie unsinnig die “Befürchtungen” bezüglich des LHC sind, gerademal eine planck-zeit, also 1.351250e-43 sekunden würde ein BH mit einer Masse von einem microramm “leben”.

  39. #39 ssssssssssss
    12. Januar 2010

    Ich glaube, ich habe es fast verstanden.

    Oben wurde gesagt, dass man an ein SL und einen Stern gleicher Masse gleich dicht heran kann.
    Es hätte die gleichen Auswirkungen, oder?

    Das würde doch bedeuten (vorausgesetzt der Stern und das SL lägen nebeneinder), dass der Ereignishorizont des SL dann im Vergleich zum Mittelpunkt des Sterns quasi unter der Sternenoberfläche läge?

    Versteht ihr, was ich meine?

  40. #40 Marek
    12. Januar 2010

    @Bullet, Christian

    Besten Dank erst mal. Muss noch etwas fürs Bruttosozialprodukt tun. Die grauen Zellen sind vorerst anders belegt. 🙂

  41. #41 Bullet
    12. Januar 2010

    @sssssssssssssss:
    Gerade das ist doch falsch. Ein SL und ein Stern gleicher Masse üben dieselbe Gravitationswirkung auf Objekte in der Umgebung aus. (Entfernung vom Mittelpunkt des SL/Sterns!!!!!!!!!!!!)
    Bei unserer Sonne kommst du aber nur knapp 750 000 km an das Zentrum ran, weil du dann auf der Sonnenoberfläche landest.
    An ein SL gleicher Größe kommst du laut Rechnung oben von Stefan aber bis auf die Entfernung von knapp 3 Kilometern ran. Und jetzt überleg dir mal den Unterschied zwischen der Anziehungskraft einer Quelle in doppelter Entfernung wie der des Mondes und einer, die nur 5 U-Bahn-Stationen entfernt ist.

  42. #42 ssssssssssss
    12. Januar 2010

    @bullet
    Ich seh schon, du hast mich nicht verstanden, denn ich wollte genau das ausdrücken, was du geschrieben hast, der Stern selbst ist einfach im Weg, beim SL ist noch Platz.

  43. #43 Bullet
    12. Januar 2010

    Aso. Dann stimmts “quasi”, ja.

    Das ist natürlich streng genommen nicht korrekt, weil bei einem normalen Stern der Ereignishorizont nicht zustandekommt. Laut Rechnung oben läge er drei Kilometer vom Sternzentrum entfernt, aber dadurch, daß an dieser Stelle fast die gesamte Sternmasse oberhalb des Ereignishorizontes liegt, ist die Gravitationskraft da eher nach draußen gerichtet. Und damit funktioniert das nicht. Deswegen muß sich die Sternmasse ja erstmal so weit komprimieren, daß alle Sternmaterie innerhalb dieses Radius liegt. Dann klappts auch mitm Loch. Dem, ähm, schwarzen.

  44. #44 Stefan
    12. Januar 2010

    Wobei – und das ist nicht unwichtig für die Diskussion – dazu gesagt werden muss, das es sich hier, in den Berechnungen auf der von mir gebrachten Webseite, um ein stark vereinfachtes BH handelt. Ein realistisches BH rotiert, bildet nicht nur einen Ereignishorizont aus, sondern auch eine Ergosphaere (Kerr-Metrik). Darin rotiert _alles_ innerhalb dieser Sphäre mit dem Schwarzen Loch mit.

    Allein das bedeutet, das sich Schwarze Löcher anders in deren Nähe verhalten, als wir in der Newton-Metrik gewohnt sind. Das sogenannte Frame-Dragging, also das “aufwuzzeln” 😉 des Raums bei den Abstand eines Sonnenradius zum Schwarzen Lochs, beeinflusst Objekte schon wohl noch mehr als in der Nähe der Sonneoberfläche. Und wahrscheinlich – auf längere Sicht gesehen – auch im Abstand Erde-Sonne.

    Alleine deswegen kann in der Nähe des Horizont nicht mehr die vereinfachte Newton-Metrik herangezogen werden, sondern es muss relativistisch gerechnet werden, um zu “vernünftige” Werte zu kommen. Aber auch wegen anderer relativistischen Phänomene gelten nicht mehr die newontischen Näherungsrechungen.
    Diese Nah-Horizont Phänomene haben für eine Erde zuerst wohl kaum eine Auswirkung, für die Stabilität der Erdumlaufbahn über längere Sicht sehrwohl.

    Hinzu kommen, bei solch nahen Begegnungen, das wir noch immer in Ungewissen sind, wie sich ein BH tatsächlich “verhält”. Im Moment gibt es für rotierende (und geladene) BH nur eine Vakuumslösungs (eine sogenannte “äußere Lösung”), eine innere Lösung, also eine Rechnung wo angenommen wird das der Horizont mit einem Fluid gefüllt ist, gibt es bislang noch nicht. Hier gibt es bestimmt Unterschiede zu der Vakuumslösung.

    Wie geschrieben, würde die Sonne augenblicklich zum Schwarzen Loch kollabieren, gäbe es vorerst (ausgenommen von Dunkelheit und Kälte ;-)) keinen Unterschied für die Erdumluafbahn. Über längere Zeiträume sollte sich aber eben auch das “nicht-Newton-Verhalten”, also das relativistische Verhalten ausserhalb des Horizonts bemerkbar machen.

  45. #45 Thomas J
    12. Januar 2010

    @Stefan

    Und wie würde sich das auf die Erdumlaufbahn auswirken?

  46. #46 Alex
    12. Januar 2010

    Also ich finde das Bild des Staubsaugers nach wie vor ziemlich passend: Alles, was dem Schwarzen Loch zu Nahe kommt, also seinen Ereignishorizont (seine Staubdüse) überschreitet, wird angesaugt und gnadenlos verschluckt. Auf weit entfernte, schwere Objekte (die Erde, der Sessel am anderen Ende des Zimmers) hat der Staubsauger dagegen kaum einen Einfluss.
    Nur die Tatsache, dass die Saugleistung mit der Zeit immer stärker wird, passt nicht so ganz zur Erfahrungswelt des gemeinen Hausmanns 😉

  47. #47 Stefan
    12. Januar 2010

    @Thomas: durch das Frame Dragging eines Schwarzen Loches entstehen auch andere – natürlich weit geringere – Gravitationspotentiale in der Nähe eines Schwarzen Loches. Das hat natürlich Auswirkung auf den Raum nicht nur in der Nähe, sondern auch weiter entfernt. Umso weiter entfernt, umso mehr gelten dann natürlich die Lösungen von Newton wieder. Aber selbst die die Umlaufbahn des Merkus wird ja durch die relativistischen Auswirkungen der Sonnengravitation beeinflusst. Aber in der Nähe der Sonne reichen noch immer die Newton-Lösungen um den Raum zu beschreiben (bis auf kleine Abweichungen), in der Nähe von Schwarzen Löcher muss aber relativistisch gerechnet werden.

  48. #48 Thomas J
    12. Januar 2010

    @Stefan

    Jo, klar, hier gings aber um das Gedankenexperiment, dass anstatt der Sonnen ein schwarze Loch mit Sonnenmasse da ist, als der konkrete Unterschied zum realen Zustand jetzt.
    Darum habe ich gefragt, was das für eine Auswirkung auf die Erdbahn hätte.
    Und wo entstehen noch diese anderen Gravitationspotentiale?

  49. #49 Stefan
    12. Januar 2010

    @Alex: “Alles, was dem Schwarzen Loch zu Nahe kommt, also seinen Ereignishorizont (seine Staubdüse) überschreitet, wird angesaugt und gnadenlos verschluckt.”

    Eben nicht, nur bei bestimmten Arten von Schwarzen Löchern. Schwarze Löcher kleiner größe saugen gar nichts mehr, sondern sie spucken nur noch.
    Heute ist die Temperatur von stellaren Schwarzen Löchern derart gering (und dmait auch ihre Strahlung), das sie die Temperatur des umgebenden Raums unterschreitet. In der Zukunft wird die Temperatur des Universums aber weiter fallen und dann schaut die Sachlage wieder anders aus. Dann beginnen Schwarze Löcher, auch grosse und wenn auch schwach, zu strahlen.

    Natürlich wird auch dann die Gravitation dafür sorgen, das Objekte angezogen werden, aber im Gegenzug dazu wird mehr Masse, in Form von Hawkingstrahlung, abgestossen – ob man da noch von saugen sprechen kann … 😉

    Kleine Schwarze Löcher und in der Zukunft auch größere, für die gilt das Bild des Staubsaugers nicht mehr – das Bild des Staubsaugers gilt nur für stellare oder größere Schwarze Löcher in der jetzigen Phase des Kosmos.

  50. #50 Stefan
    12. Januar 2010

    @Thomas: die zusätzlichen Gravitationspotentiale entstehen innerhalb der Ergosspähre. Es gibt Berechnungen, das selbst diese so dicht werden können, das sie selbst Singluaritäten bilden können und den eigentlichen Horizont “wegsaugen” könnten, so das eine “nackte Singluarität” entstehen kann.

    Wobei, nach der kosmischen Zensur von Hawking ist das nicht ohne weiteres möglich und Berechnungen zeigen, das sich sofort einer neuer Horizont bildet, der die Singluarität verdeckt.
    “Weggesaut” werden kann der Ereignishorizont deswegen, weil der Horizont eines Schwarzen Lochs auch ein tatsächlisches physikalisches Objekt im eigentlichen Sinne darstellt.

  51. #51 Bullet
    12. Januar 2010

    “weggesaut”? *g*

  52. #52 Stefan
    12. Januar 2010

    “weggesaut” ist tatsächlich gut und irgendwie auch passend, vielleicht hält der Begriff ja einzug in die Fachwelt, es gibt eh soviele seltsame “eingeenglischte” deutsche Wörter. 😉

  53. #53 MartinB
    12. Januar 2010

    Ein Kommentar noch zum Satz

    Erst ab dem Ereignishorizont wird’s gefährlich.

    Da kommen noch die Gezeitenkräfte dazu: Weil die Anziehung des Schwarzen Lochs ja mit dem Quadrat des Abstands abnimmt, wird ein hinreichend großes objekt, das sich dem SL näher, zerrissen, weil der Teil der dem Loch näher ist, viel stärker angezogen wird.
    Ganz so dicht geht man also doch besser nicht ans SL ran, denn man ist nicht einfach im freien Fall sondern wird ziemlich böse zerrupft.

  54. #54 H.M.Voynich
    12. Januar 2010

    @MartinB:
    Es sei denn, das SL ist sehr sehr sehr sehr groß – dann werden die Gezeitenkräfte klein.

  55. #55 Stefan
    12. Januar 2010

    @Martin B: hier ist interessant, das bei Schwaren Löchern die groß genug sind (mehrere millionen Sonnenmassen) die Gezeitenkräfte so gering werden, das eine Überquerung des Horizonts lebend möglich wäre. Was dann geschieht ist wohl der Traum jeder/s TheoretikerIn, nur das dieseR kein Paper mehr verfassen kann … 😉 Allerhand exotisches Zeug würde man dann zu Gesicht bekommen, Zeitdimensionen die “raumartig” werden und wenn man nur gut genug reinplumst und es schafft den Torus (der ja bei einen rotierenden Schwarzen Loch die Punktsingularität ersetzt) zu durchqueren, das dann Zeitreisen möglich werden und vieles spannendes mehr. 😀

    @Informationparadoxon: Das ist ja auch mehr als spannend, wie Florian geschrieben hat, bedeutet ein Verschwinden von Materie eine Abnahme der Entropie im restlichen Universum – Schwarze Löcher müßen daher selbst eine Entropie besitzen, daher haben sich auch eine Temperatur, daher haben sie auch eine Strahlung – der Denkweg zur Hawkingstrahlung.
    Die Entropie der Informationstheorie ist nun nahe verwandt mit der thermodynamischen Entropie und so einfach kann Information (auch wenn “Information” physikalisch nur schwer fassbar ist) auch nicht verschwinden. Und Materie die in ein Schwarzes Loch fällt besitzt nun mal Information.

    Was geschieht nun mit der Information? Hawkingstrahlung ist eine Wärmestrahlung und besitzt kaum Information, bis gar keine Information.
    Mit dieser Überlegung kommt man zu der HolographischenTheorie von t’Hooft, wonach sich Vorgänge innerhalb eines Systems gleichwertig an dessen Oberfläche beschreiben lassen. Also physikalische Phänomene innerhalb des Schwarzen Loches lassen sich gleichwertig auf dessen Oberfläche beschreiben – so die Theorie und damit löst sich auch das Informationsproblem.
    Und deswegen die Informationsmenge auch nicht – wie klassisch und in der Quantenmechanik beschrieben – abhängig vom Volumen, sondern von dessen Oberfläche.

    Das holographische Modell hat gewisse Vorteile, angewendet auf das Universum (wobei: wo ist hier die Grenze? Was ist diese? Gibt es überhaupt eine? bisher beziehen sich wissenschaftl. Papers auf Schwarze Löcher) können physikalische Phönomene im 3-Dimensionalen Universum gleichwertig auf dessen 2-Dimensionalen Oberfläche beschrieben werden (eine Oberfläche hat immer eine um 1 niedrigere Diemension als das Objekt selbst).
    Der Vorteil dieser Beschreibung ist nun, das bei dieser Berechnungsart die Gravitation wegfällt. Gravitative Ereignisse in einer 3-D Welt können also auf der 2-D Oberfläche beschrieben werden, als gäbe es gar keine Gravitation. Die Gravitation “entsteht” also nur, wenn wir uns in die 3-D Welt begeben.

    Dieser Gedankengang muss nun noch auf die String-Theorie erweitert werden (was auch fleissig geschieht) und vorallem sind hier Überlegungen im Zusammengang des Randall-Sundrum-Modell ( https://en.wikipedia.org/wiki/Randall%E2%80%93Sundrum_model ) interessant.

  56. #56 ZielWasserVermeider
    12. Januar 2010

    Hmmm… wenn ich das alles so lese:

    Wenn ein Stern zum Schwarzen Loch wird, und er einige Planeten hat, die die Supernova überstanden hätten, könnte es dann Systeme mit Planeten geben, die ein schwarzes Loch umkreisen?(oder sie wurden später eingefangen, oder aus den Kometen und Trümmern gebildet)

    Könnten sich also Schwarzelochsysteme mit Planeten bilden…. oder ist die Schockwelle der Nova und die sich ausbreitenden Gase zu stark?

    Gruß
    Oli

    P.S.: wäre doch ne nett Vorstellung, daß es da draussen solche Systeme gäbe 😉

  57. #57 Bullet
    12. Januar 2010

    doch, es soll sowas geben… *malirgendwiegehörthab*

  58. #58 cimddwc
    12. Januar 2010

    @ZielWasserVermeider:

    In Spektrum der Wissenschaft 10/09 stand etwas über entdeckte Exoplaneten um Neutronensterne und Braune und Weiße Zwerge – so manche Explosion können Planeten demnach schon überleben oder sich aus den übrigbleibenden Trümmern, die sich erst in einer Scheibe sammeln, hinterher wieder bilden.

    SL kamen dabei aber nicht zur Sprache. Eine interessante Idee ist es auf jeden Fall…

  59. #59 MartinB
    12. Januar 2010

    @Stefan
    “Was dann geschieht ist wohl der Traum jeder/s TheoretikerIn, nur das dieseR kein Paper mehr verfassen kann ”
    Würde auch nix nützen, denn durch die gravitative Zeitdilatation braucht er vom Standpunkt des Publikationsverlages aus ja unendlich lange, den Horizont zu durchqueren…

  60. #60 Alex
    12. Januar 2010

    Wie kann man denn ein erzeugtes Schwarzes Loch wieder zerstören?
    Irgendwie müsste doch immer etwas dadurch “fließen”, aber wohin?
    Kann man eine Kamera durchschicken, die dann Bilder von der anderen Seite überträgt (über den Raum, nicht über das SL; das geht ja nicht)?

  61. #61 ZielWasserVermeider
    12. Januar 2010

    @cimddwc & Bullet

    Unser Weltraum soll ja ca. 13,7 Milliarden Jahre alt sein(abzüglich der ersten paar hundert Millionen Jahre).
    Da wäre ja genügend Zeit, daß sich in den Jahren auch um ein schwarzes Loch eine Planetensystem gebildet haben könnte.

    Massenreiche Stern wird zum schwarzen Loch…. später gerät das daraus entstehende SL in eine Gas und trümmerreiche/gasreiche Region…. z.B. Pferdekopfnebel(ist doch ne Babystation für Sterne …oder?)…. Und Gasnebel in denen auch sehr schnellebige Sterne entstehen und Vergehen(z.B. zu SL werden) könnten dann auch genügend Gase hin und her schubsen, daß sich um ein SL eine rotierende Gasscheibe bildet und daraus ein Planetensystem.

    Das Hubble-Teleskop(siehe ESO/Hubble-Casts) hat ja wohl zahlreiche Scheiben in Gaswolken entdeckt, wo sich neue Systeme/Sonnen bilden….. in wieweit würde sich dann ein SL(mit entstehenden Planeten) -System davon unterscheiden?
    ?
    Oli

  62. #62 Marek
    12. Januar 2010

    @Christian W

    Das mit dem Loch ist doch die bessere Analogie. 🙂
    Mann, mann, das hat mal wieder gedauert, bis ich´s gerafft habe. Danke auch die anderen.

  63. #63 MartinB
    12. Januar 2010

    @Alex
    “Kann man eine Kamera durchschicken, die dann Bilder von der anderen Seite überträgt”
    Nee, kann man nicht. Selbst wenn die Gezeitenkräfte sie nicht zerstören, dann braucht sie aus unserer Sicht doch unendlich lange, um den Ereignishorizont zu erreichen, weil die extreme Gravitation zu einer Zeitdilatation führt.

    In schwarze Löcher reinfliegen kann nur Perry Rhodan durch die Schwarzen Sternstraßen, aber die Archäonten hatten ja auch schwer was los, als sie die gebaut haben. 😉

  64. #64 Marek
    12. Januar 2010

    … an die anderen.

  65. #65 ZielWasserVermeider
    12. Januar 2010

    Mal abgesehen von meiner Fantasie….

    Gibt es irgendwelche Links(also wissenschaftliche Seiten) zu SL + Planeten? …. oder Spekulationen dazu?

    ?
    Gruß
    Oli

  66. #66 H.M.Voynich
    12. Januar 2010

    “… dann braucht sie aus unserer Sicht doch unendlich lange, um den Ereignishorizont zu erreichen, weil die extreme Gravitation zu einer Zeitdilatation führt.”

    Mhhh – wie ist das denn eigentlich: wenn alles spätestens hinterm Ereignishorizont aus unserer Sicht still stehen bleibt, festgefroren in der Zeit, dann gibt es doch eigentlich gar keine Singularität im Zentrum, bzw. erst nach unendlich langer Zeit? Zum Verdampfen benötigt das SL jedoch nur eine endliche Zeit (sei sie auch noch so groß). Ergo kommt es nie zur Singularität?

  67. #67 mein echtes ich
    13. Januar 2010

    Hier ein parr sehr leichtverständliche erklerunhgen über schwarzelöcher die viele Fragen beantworten kann:-)

    https://www.youtube.com/watch?v=3QXzFpcjqwQ

  68. #68 MartinB
    13. Januar 2010

    @HMVoynich
    Bin ich mir nicht sicher, aber ich glaube, der Unterschied besteht darin, dass im einen Fall die Materie, die zum SL wird, ja schon “innerhalb” des Horizonts ist, wenn er sich bildet, während sie im anderen Fall nachgeliefert wird. Das ist aber nur SPekulation.

  69. #69 Stefan
    13. Januar 2010

    “… dann braucht sie aus unserer Sicht doch unendlich lange, um den Ereignishorizont zu erreichen, weil die extreme Gravitation zu einer Zeitdilatation führt.”

    in dieser Erklärung steckt eh schon die Lösung: nur für aussenstehende Beobachter dauert der Fall unendlich lange, für den oder die TheoretikerIn, die (unnötiger weise) Paper schreibend sich ins Schwarze Loch stürtzt, vergeht die Zeit “normal”.

    Ansonsten könnten Schwarze Löcher ja überhaupt nicht mehr durch Materiezuwachs wachsen.

  70. #70 Bjoern
    13. Januar 2010

    @Stefan:
    “Ansonsten könnten Schwarze Löcher ja überhaupt nicht mehr durch Materiezuwachs wachsen.”

    Ich denke, was man hier auch berücksichtigen müsste, ist, dass die normale Beschreibung von SL durch die Schwarzschild-Metrik *statisch* ist, d. h. diese Beschreibung kann schon rein durch ihren Ansatz eine Änderung von SL gar nicht beschreiben!

    Hat jemand eine Ahnung, ob jemand sich schon mal an einer zeitabhängigen Variante der Schwarzschild-Metrik versucht hat?

  71. #71 Bjoern
    13. Januar 2010

    @Florian: “Das, was ein schwarzes Loch so außergewöhnlich macht, ist eine andere Eigenschaft: seine Dichte!”

    Das stimmt so streng genommen auch nicht – ein Objekt ist nicht automatisch ein SL, nur weil es eine große (mittlere) Dichte hat. Umgedreht: auch Objekte mit kleiner (mittlerer) Dichte können trotzdem ein SL sein! Z. B. hätte ein SL mit etwa 130 Millionen Sonnenmassen (und so was kommt in Zentren von Galaxien durchaus vor) gerade mal die mittlere Dichte von Wasser… (1 g/cm^3) … wenn ich mich nicht verrechnet habe 😉

    Was bei einem SL groß sein muss, ist weder die Masse noch die Dichte – sondern der Quotient aus Masse und Radius, oder, äquivalent, das Produkt aus Radius zum Quadrat und der Dichte.

    Sieht man rechnerisch sehr einfach: ein Objekt ist ein SL, wenn sein Radius R kleiner als sein Schwarzschild-Radius ist. Also R muss kleiner sein als (2 G M) / c^2, wobei G die Gravitationskonstante, M die Masse und c die Lichtgeschwindigkeit ist. Umstellen liefert: M/R muss größer sein als c^2 / (2 G). Das wäre die gesuchte Bedingung.

  72. #72 H.M.Voynich
    13. Januar 2010

    @stefan:
    “in dieser Erklärung steckt eh schon die Lösung: nur für aussenstehende Beobachter dauert der Fall unendlich lange, für den oder die TheoretikerIn, die (unnötiger weise) Paper schreibend sich ins Schwarze Loch stürtzt, vergeht die Zeit “normal”.”

    Die Hawking-Zerstrahlung dauert aber – ebenfalls für den außenstehenden Beobachter – nur endlich lange.
    Für den ins Loch stürzenden zeitdilatierten Theoretiker dürfte das SL schon wieder zerstrahlt sein, bevor er auch nur mit der Wimper zuckt, was das Schreiben eines Papers zusätzlich erschweren dürfte?

  73. #73 Stefan
    13. Januar 2010

    @Bojem: die Schwarzsschild-Metrik ist doch nur deswegen “statitisch”, weil sie ein nicht rotierendes Schwarzes Loch beschreibt, Massenzuwächse/abnahmen sind von dem “statisch” sein doch gar nicht betroffen.
    Die SSM ist ein Spezialfall für ein nicht-rotierendes und nicht geladenes Schwarze Loch – mehr nicht. Wie die Kerr-Newman-Metrik rotierende und geladene Schwarze Löcher beschreibt, aber jetzt nicht speziell “dynamisch” im Sinne einer Masseänderung.

    @H.M.Voynich: Bei der Hawkingstahlung geht es auch um (relativistische) quantenmechanische Prozesse. Die bisherige klassischen (in dem Fall: nicht quantenmechanische) Beschreibungen würde das gesagte gelten, aber hier kommt eben die Quantenfeldtheorie ins Spiel oder anders gesagt eine relativistische Form der QF-theorie.

  74. #74 Stefan
    13. Januar 2010

    @Bojem: “Hat jemand eine Ahnung, ob jemand sich schon mal an einer zeitabhängigen Variante der Schwarzschild-Metrik versucht hat?”

    Ja, das ist aber nur mittels komlizierter Störrechnungen als Näherungslösungen möglich.

    Und um vielleicht das Vorposting aufzugreifen, vielleicht liegt dahingehend das MIssverständnis: Natürlich ist die Schwarzschildmetrik eine stationäre Lösung in dem Sie eine Zustandsbeschreibung ist, aber ich kann mit ihr doch Zustandsänderungen beschreiben.

  75. #75 Bjoern
    13. Januar 2010

    @stefan: Wie genau willst du mit einer Schwarzschild-Metrik denn beschreiben, wie sich irgendein Objekt dem SL nähert, von diesem “verschluckt” wird und dadurch sich die Masse des SL erhöht? Genau darum ging’s hier schließlich eigentlich.

    Sicher könnte man einfach sagen, dass man vor der Zeit, zu der das Objekt (Masse m) verschluckt wird, eine Schwarzschild-Metrik mit Masse M verwendet, und danach eine Schwarzschild-Metrik mit Masse M+m – aber das wäre ja wohl nur eine sehr grobe Näherungslösung und alles andere als eine genaue Beschreibung des Vorgangs.

  76. #76 Bjoern
    13. Januar 2010

    @Stefan: Dass die Lösung *statisch* ist, heißt, dass sie nicht von der Zeit abhängt. Also: keine Änderungen finden statt. Das schließt nicht nur Rotation aus, sondern auch Massenänderungen o. ä.

    Schlägst du vor, dass man in der Schwarzschild-Metrik einfach M durch M(t) ersetzt, und dass das dann plötzlich auch Vorgänge beschreibt, bei denen sich die Masse ändert? Ich möchte stark bezweifeln, dass das dann immer noch eine Lösung der Einstein-Gleichungen wäre (gebe zu, ich hab’s bisher nicht durchgerechnet 😉 ).

    Oder wie soll das deiner Ansicht nach funktionieren?

  77. #77 Andreas
    13. Januar 2010

    Danke Florian, für diesen Beitrag.

    Mich nervt es jedesmal, wenn die Medien von Schwarzen Löchern berichten, die alles aufsaugen. Im Englischen gibt’s leider auch die „sucking black holes“. (Deshalb auch der Abschnitt „Vorsicht: Hier saugt keiner!“ unter https://www.wissenschaft-online.de/astrowissen/lexdt_a02.html#akk bzw. S. 33 im Buch) 🙂

    Zur Frage wie man Schwarze Löcher zerstören kann:
    Man kann die Massenenergie und ggf. Rotationsenergie eines Schwarzen Loches durch Penrose-Prozesse abführen (siehe https://www.wissenschaft-online.de/astrowissen/lexdt_p02.html#penrose ). Das ist ein Effekt der im Rahmen der Allgemeinen Relativitätstheorie und verständlich ohne Quantentheorie.
    Mit Quantentheorie (nur quantisierte Teilchen/Felder nötig, aber keine quantisierte Gravitation) kann man den Zerfall eines Schwarzen Loches durch Hawking-Strahlung erklären. Siehe https://www.wissenschaft-online.de/astrowissen/lexdt_h02.html#hawk

    Zur Dichte eines Schwarzen Loches:
    Vom Standpunkt der Allgemeinen Relativitätstheorie haben alle Löcher die gleiche Dichte, nämlich unendlich, die in der Singularität bei Radius r=0 angenommen wird. Man darf nicht die Lochmasse über die durch den Ereignishorizont gebildete Kugel verteilen! In der Schwarzschild-Lösung ist die Raumzeit im Prinzip leer, denn es ist eine Vakuumraumzeit mit Energie-Impuls-Tensor T=0. Die Lochmasse steckt in der Singularität und nur dort.

    Zur Schwarzschild-Lösung und deren Zeitabhängigkeit:
    Die Schwarzschild-Lösung ist einzig allein beschrieben durch einen zeitunabhängigen Massenparameter M. Sie ist statisch und kugelsymmetrisch. Man kann Sie nicht einfach zeitabhängig zu einem M(t) machen, weil dass die Symmetrien dieser Raumzeit verändern würde – das Wesen der Schwarzschild-Lösung wäre verändert.

    Man kann die Schwarzschild-Metrik als Anfangsraumzeit vorgeben und weitere Massen damit in Wechselwirkung treten lassen. Anschaulich würde der Trichter oben deformiert werden durch die Krümmung der Raumzeit weiterer Massen. Dann muss man die numerische Allgemeine Relativitätstheorie betreiben. Die Dynamik wird natürlich bestimmt durch die Einstein-Gleichungen G = T.

    Man mag einwenden, dass die Schwarzschild-Lösung dann ja gar nicht in der Natur realisiert sein kann. Nun, aber sie beschreibt die Verhältnisse näherungsweise recht gut, wenn die Massen in der Umgebung klein sind. Relativisten beschreiben sogar die relativistische Lichtablenkung an der (sogar rotierenden!) Sonne mit der Schwarzschild-Lösung. Man muss hierbei nicht gleich die Kerr-Lösung benutzen, weil die Sonne recht langsam rotiert. (Anmerkung: Bei schnell rotierenden Neutronensternen funktioniert das nicht mehr und sie haben ganz eigene Raumzeiten, wie z.B. die Manko-Lösung, beschrieben durch vier Parameter Masse, Drehimpuls, Massenquadrupolmoment und magnetisches Moment.)

    Es gibt keine zeitabhängige Variante der Schwarzschild-Lösung. Dennoch kann man zeitabhängige Effekte, zum Beispiel das Fließen einer Flüssigkeit, in der Schwarzschild-Metrik berechnen und dabei annehmen, dass die Metrik als Hintergrund fungiert und selbst nicht von der Flüssigkeit deformiert wird („als Hintergrund fungiert“ heißt, dass die bleibt, wie sie ist Schwarzschild-Metrik – also auch die Masse M bleibt konstant). Das ist nicht ganz selbstkonsistent im Rahmen der Allgemeinen Relativitätstheorie, aber es ist eine vertretbare Näherung. Mit dieser Methode betreibt man Akkretionstheorie in der Umgebung Schwarzer Löcher. Das ist das Gebiet der General Relativistic Magnetohydrodynamics, also das Studium von magnetisierten Flüssigkeiten vor dem Hintergrund der Schwarzschild- oder Kerr-Raumzeit. Damit lässt sich simulieren, wie Schwarze Löcher Materie verschlingen und dabei Jets herausschießen

    Beste Grüße,
    Andreas (KOSMOlogs)

  78. #78 Dennis S.
    14. Januar 2010

    Jetzt muss ich doch mal nachfragen.
    Hat ein Schwarzes Loch einen unendlich hohe Dicht und ist somit unendlich klein, oder hat ein Schwarzes Loch, in Florians beispiel unsere Sonne(3 km), eine bestimmte Dichte und hat somit einen bestimmten Durchmesser? Falls das Erste stimmt, dann wäre der Ereignishorizont unserer Sonne 1,5km vom Zentrum entfernt.

    Also wie ist es denn? 🙂

    ps. Super Arbeit Florian. Allein die 2012 Aufarbeitung ist einen Preis wert.

  79. #79 Stefan
    14. Januar 2010

    @Bojem: okay, die klassischen Lösungen der Schwarzschildmetrik sind nicht zeitabhaenging, trotzdem kann ich Zustandsänderungen beschreiben, ich kann sie nicht zeitabhängig beschreiben, aber ich kann ihre einzelnen Zustände beschreiben.
    Vorallem weil es ja inzwischen (schon sehr lange, kurz nach der klassischen Lösung von Schwarzschild) auch sogenannte innere Lösungen der Schwarzschildmetrik, wo als das innere eines SL als ein Fluid beschrieben wird und das ist natürlich zeitabhängig, im Gegensatz zur sehr einafchen und klassischen Vakuumslösung der Schwarzschildmetrik.

    Und naürlich gibt es zeitabhängige Modelle der Black-Branes oder Blach-Strings, wie Schwarze Löcher in der Stringtheorie genannt werden oder zb auch die Meyers-Perry-Solution, also ein 5-Dimensionales Schwarze Loch – hier müssen Störungsrechnungen zur Hilfe genommen werden (also eine Näherungslösung).

    Das ist halt das Problem wenn wir hier von dem einfachsten Modell eines Schwarzen Loches sprechen wollen und uns keine Papers um die Ohren hauen wollen, aber wie geschrieben: ich kann auch die einzelnen Zustände eines Schwarzen Loches mit der Schwarzschild-Metrik bestimmen und gravitative Veränderungen in der Nähe eines Schwarzen Loches ebenfalls mittels Näherungslösungen bestimmen, das ist ansich nichts ungewöhnliches.

    @Dennis: ein klassisches Schwarzes Loch hat unednlich hohe Dichte und ist unendlich klein, nicht mehr ganz so kalssisch betrachte SL haben Planck-Größe und planch-Dichte.

  80. #80 Stefan
    14. Januar 2010

    ich bemerke gerade das Andreas das schöner beschreiben hat und ich mir mein geschreibsel ersparen hätte können. 😉
    danke für die schöne Erklärung!

  81. #81 Bjoern
    14. Januar 2010

    @Andreas, @Stefan: O. k., die Beschreibung geht also nur näherungsweise (hätte ich auch vermutet…).

    Aber was kommt denn jetzt konkret beim interessanten Problem raus: wir betrachten eine Testmasse, die auf das SL zu fällt und schließlich von ihm verschluckt wird. Bekanntlich dauert das für einen äußeren Beobachter unendlich lange; trotzdem misst der äußere Beobachter aber auch, dass die Masse des SL mit der Zeit zunimmt – eben weil Dinge verschluckt werden. Wie passt das zusammen? Wie kann man messen, dass die Masse des SL durch Akkretion zunimmt, obwohl man gleichzeitig auch misst, dass das Verschlucken von Objekten unendlich lange dauert?

    Was sagt da die näherungsweise Rechnung dazu? Dauert das Verschlucken denn nun unendlich lange oder nicht? Oder sieht es für den Beobachter nur so aus, als ob das Verschlucken unendlich lange dauern würde, in Wirklichkeit geht es aber in endlicher Zeit vor sich?

  82. #82 Bullet
    14. Januar 2010

    Björn: >blockquote>wir betrachten eine Testmasse, die auf das SL zu fällt und schließlich von ihm verschluckt wird. Bekanntlich dauert das für einen äußeren Beobachter unendlich lange; trotzdem misst der äußere Beobachter aber auch, dass die Masse des SL mit der Zeit zunimmt – eben weil Dinge verschluckt werden. Wie passt das zusammen?
    Das ist ein Gedankenexperiment. Und die Logik sagt, es müsse so sein.
    Faktisch ist nichts davon aber jemals beobachtet worden. Nicht, weil stattdessen etwas ganz anderes geschieht, sondern weil wir noch viel zu wenig Beobachtungsdaten und -möglichkeiten haben. Die Frage ist wirklich interessant, keine Frage, aber afaik mit empirischen Daten z.Z nicht klärbar. Oder weiß da jemand mehr?

  83. #83 Andreas
    14. Januar 2010

    @Björn
    Der Außenbeobachter kann nur indirekt feststellen, dass das Schwarze Loch gewachsen ist, nämlich indem er beobachtet, dass die Größe des „schwarzen Flecks“, den der Ereignishorizont im Kosmos hinterlässt, größer geworden ist. Den Horizont selbst (eine Fläche mit exakt verschwindender Emission nach der ART) kann ein Außenbeobachter aber nicht sehen, weil er für alle Außenbeobachter in dessen Zukunft liegt. Warum ist das so? Das lässt sich anhand von Raum-Zeit-Diagrammen zeigen. Hier schneiden raumartige Hyperflächen mit konstanter Zeit, auf der ein Außenbeobachter liegt, niemals den Horizont – sorry, aber das ist fachsprachliche Ausdrucksweise für „liegt in der Zukunft des Außenbeobachters“.

    Das sagt uns übrigens auch, dass man niemals mit elektromagnetischer Strahlung die Existenz eines Ereignishorizonts beweisen kann! Das ist ein echtes Dilemma in der Astronomie, das wohl nur mit der Beobachtung von Gravitationswellen lösen kann.

    Man kann aktuell klassische Schwarze Löcher vom Schwarzschild-Typ nicht eindeutig nachweisen bzw. sie nicht von alternativen Szenarien wie Gravastern oder Holostern unterschieden. Das Vertrauen in die klassischen Schwarzen Löcher ist so hoch, weil man astronomisch gemessen hat, dass viele (Kandidaten für) Schwarze Löcher rotieren. Dies wird mit der Kerr-Lösung in der ART beschrieben und für die Kerr-Lösung gibt es bislang keine rotierende Alternative, die ebenfalls rotierende Schwarze Löcher gut beschreibt. Die Kerr-Lösung erklärt auch gut die Entstehung der beobachteten relativistischen Jets. Schwarzschild, Grava- und Holostern können da nicht mithalten.

  84. #84 Bjoern
    14. Januar 2010

    @Andreas:
    “Der Außenbeobachter kann nur indirekt feststellen, dass das Schwarze Loch gewachsen ist, nämlich indem er beobachtet, dass die Größe des „schwarzen Flecks“, den der Ereignishorizont im Kosmos hinterlässt, größer geworden ist.”

    Oder indem er feststellt, dass die Gravitation des SL (in gleicher Entfernung zum Zentrum) größer geworden ist, würde ich sagen.

    Das beantwortet meine Frage aber immer noch nicht: dauert das Verschlucken und damit das Anwachsen des SL nun unendlich oder endlich lange?

    Grava- und Holostern hatte ich übrigens auch noch nie gehört. Ersteres habe ich gerade bei Wikipedia nachgelesen; zu zweiterem steht da aber leider nix. Was ist ein Holostern?

  85. #85 Andreas
    14. Januar 2010

    @Björn
    Richtig, eine Zunahme der Gravitation bei gleichem Abstand kann für eine Zunahme der Zentralmasse sprechen. In der Praxis misst man z.B. Bahnabstand und Bahnperiode eines Orbiters und wendet das 3. Kepler-Gesetz an, um die Zentralmasse zu bestimmen (prominenter Fall: Stern S2 um Zentrum der Milchstraße Sgr A*). Dann schon von einem Wachstum des Schwarzen Loches zu sprechen ist in den meisten Fällen recht kühn, weil die Orbits viel weiter vom Ereignishorizont weg sind.

    Doch Björn, die Frage ist beantwortet: Für einen Außenbeobachter dauert es unendlich lange, bis ein Testteilchen den Ereignishorizont erreicht, denn – wie gesagt – der Ereignishorizont liegt immer in der Zukunft.

    Dennoch kann ein Außenbeobachter die dunkle Zone in der Nähe des (nicht am) Ereignishorizont wahrnehmen und aus der Größe dieser Dunkelzone auf die Lochmasse schließen. Je größer der “Schatten” des Loch, desto größer seine Masse.

    Zum Holostern:
    https://www.wissenschaft-online.de/astrowissen/lexdt_h04.html#holo
    Manche Sachen stehen nicht mal in Wikipedia 😉

  86. #86 Bjoern
    15. Januar 2010

    @Andreas: ‘tschuldigung, aber ich kapiere es immer noch nicht. Wenn es wirklich unendlich lange dauert, bis etwas reinstürzt – wie kann ein SL dann überhaupt wachsen?

    Man könnte natürlich sagen, dass das SL selbst nicht größer wird, sondern sich außen rum, kurz außerhalb des Ereignishorizonts, einfach immer mehr ansammelt, so dass es von weiter weg so wirkt, als ob das SL größer geworden wäre und eine höhere Gravitation hätte als vorher – aber wenn sich das Zeug nur außen rum ansammelt und nie wirklich reinstürzt, hat man insgesamt wohl keine kugelsymmetrische Massenverteilung mehr… was auch durch Messungen feststellbar sein müsste!

    P. S.: Danke für den Link zum “Holostern”! 🙂

  87. #87 ssssssssssss
    15. Januar 2010

    zu stefan 12.01.2010 ,14:51 Uhr
    (eine Nummerrierung der Kommentare würde das Verweisen erleichtern)

    Stefan schrieb, dass in dem SL, das unsere Sonne erzeugen würde, die Temperatur von – 273,1500 Grad Celsius herrschen würde.
    Das ist ja schon recht frisch.
    Wikipedia schreibt: “Der Nullpunkt der Kelvinskala liegt am absoluten Nullpunkt bei −273,15 °C.
    Diese Temperatur ist jedoch nach dem Nernst’schen Wärmesatz nicht mess- und erreichbar, da Teilchen bei 0 K keine Bewegungsenergie hätten.”

    Warum ist da so kalt, weil es keine Wärmequellen gibt? Sonne ist ja aus.

    Oder, meine laienhafte Theorie, wegen der extremen Dichte sind die Teilchen so dicht aneinander gepackt, dass sie sich nicht mehr bewegen können?

  88. #88 Bjoern
    15. Januar 2010

    @ssssssss (oder ssssssssssssssss?) 😉
    Stefan hat nicht geschrieben, dass diese Temperatur *in* dem SL herrschen würde – diese Temperatur hätte das SL selbst, also gewissermaßen seine “Oberfläche” – da, wo die Wärmestrahlung abgegeben wird (ich bezweifle, dass das Konzept einer Temperatur im Inneren eines SL überhaupt Sinn ergibt…). Begründung: jedes SL gibt bekanntlich Hawking-Strahlung ab; aus der Menge an abgegebener Strahlung erhält man die Temperatur des SL (ein SL ist ja wohl der idealst mögliche Schwarzkörper-Strahler… 😉 ).

  89. #89 ZielWasserVermeider
    16. Januar 2010

    @Florian Freistetter

    Huhu…. noch mal kurz meine Frage:
    Gibt es Hinweise über Planeten, die um eine schwarzes Loch kreisen, oder sich gar darum gebildet haben könnten.
    Oder ist ein Planetensystem, daß um eine Sternenleiche kreist eher nicht möglich.

    Auf die schnelle hab ich im Netz nichts gefunden.
    Für Hinweise oder Links wäre ich sehr dankbar 🙂

    Gruß
    Oli

  90. #90 Florian Freistetter
    16. Januar 2010

    @zielwasservermeider: Nein, solche System hat man bis jetzt weder entdeckt noch vermutet. Prinzipiell möglich wärs aber.

  91. #91 ZielWasserVermeider
    16. Januar 2010

    @Florian Freistetter

    Danke!
    “Prinzipiell möglich wärs aber”… falls die Astronomie mal so ein System entdecken würde…
    bitte Denkt an mich und nennt es “ZielWasserVermeiders” System 🙂

    Noch ein kleiner Tip: Du willst doch Video scheiden und aufbereiten(Blog oder Youtube) .

    Von Ubuntu gibt es die Version UbuntuStudio…. mit allem was der Mensch zum schneiden braucht.(kostet nichts.)

    Kannst dir ja auch mal die YT-Channel von AndromedasWake, SiriusStarGazing etc. angucken.

    Gruß
    Oli

  92. #92 Andreas
    17. Januar 2010

    @Björn
    Die unendliche lange Dauer bezieht sich auf eine Messung, die versucht, etwas bis zum Horizont zu verfolgen.
    Das Wachstum Schwarzer Löcher kann man trotzdem nachverfolgen, weil wir das Gebiet vor dem Ereignishorizont ja überschauen können – und dieses Gebiet wird größer. Im Hier und Jetzt können wir sozusagen einen “Schatten” des Schwarzen Loches sehen, der durch Materie-/Energieeinfall größer wird.

    Beste Grüße,
    Andreas

  93. #93 Bjoern
    17. Januar 2010

    @Andreas: Also wie ich meinte – quasi nicht das SL selbst wird größer, sondern drum herum sammelt sich eben was an?

  94. #94 Andreas
    17. Januar 2010

    @Björn
    Schau Dir mal das Titelbild unter folgender Website an: https://www.wissenschaft-online.de/astrowissen/astro_sl.html
    Das ist eine (auf der ART beruhende) Computersimulation, wie ein Schwarzes Loch aus einigem Abstand aussieht. Das Schwarze Loch sieht man klar in der Mitte als dunkle Region – dort ist auch der Ereignishorizont. Das blau-weiß leuchtende um das Schwarze Loch herum ist eine rotierende Materiescheibe. Nur weil diese leuchtet, können wir das Loch überhaupt erkennen.
    Diese schwarze Region wird größer für ein schwarzes Loch größerer Masse, d.h. sie wird anwachsen, wenn das Loch Materie/Energie aus der Umgebung aufsammelt. Bei bekannter Entfernung eines Schwarzen Loches und gemessener scheinbarer Größe der schwarzen Region folgt die Lochmasse.

  95. #95 Bjoern
    17. Januar 2010

    @Andreas: Danke, ein Bild sagt mehr als tausend Worte… 😉 Aber ist diese schwarze Region denn nun identisch mit dem Ereignishorizont, oder ist der noch weiter innen?

  96. #96 Andreas
    17. Januar 2010

    @Björn
    Der Horizont ist weiter innen, weil er wie gesagt in der Zukunft des Außenbeobachters liegt.

  97. #97 Bjoern
    17. Januar 2010

    @Andreas: Danke. Also besteht die schwarze Region sowohl aus dem SL selbst als auch aus Zeug, das außenrum fliegt, das wir aber nicht sehen können? Wenn ja: warum sollte die Massenverteilung in dieser schwarzen Region auch kugelsymmetrisch sein?

  98. #98 Andreas
    17. Januar 2010

    @Björn
    Ja.
    Die Massenverteilung in der Nähe des Lochs muss nicht kugelsymmetrisch, nicht mal symmetrisch sein; beim Bild ist sie achsensymmetrisch (eine dünne Materiescheibe). Die Metrik des Lochs wird als Hintergrundmetrik aufgefasst, d.h. die Krümmung verändert sich nicht durch die Gravitation der Scheibe. Im Bild ist eine (achsensymmetrische) Kerr-Metrik angenommen worden, d.h. das Loch rotiert. Es könnte aber ebenso gut eine Schwarzschild-Metrik benutzt werden.

  99. #99 Stefan
    17. Januar 2010

    Im Bild ist eine (achsensymmetrische) Kerr-Metrik angenommen worden, d.h. das Loch rotiert.

    Dabei sind doch bekanntlich “Beide Relativitätstheorien [..] daran gescheitert” Rotationsbewegungen zu behandeln 😉

  100. #100 Bjoern
    17. Januar 2010

    @Andreas: Nochmals danke. Dann war also meine ursprüngliche Vermutung (15.1., 16:37 Uhr) richtig…

  101. #101 Andreas
    17. Januar 2010

    @Björn
    Nicht ganz, denn es sammelt sich nicht “außen herum mehr an” 🙂 Wir werden das Material zwar nicht bis zum Schluss hineinstürzen sehen, aber dennoch signalisiert uns das gewachsene, dunkle Gebiet vor dem Horizont, das uns per Beobachtung zugänglich ist, dass das Loch gewachsen sein muss.

    Stell es Dir so vor: Ein Stern nähere sich einem Schwarzen Loch und falle hinein. Wir sehen wir der Stern röter und dunkler wird – und schauen wir auf die Uhr, so dauert es immer länger bis der Stern hineinstürzt, obwohl er doch dem Loch schon so nahe ist. Plötzlich ist der Stern so dunkel, dass er aus unserer Sicht verschwindet. In dem Moment kreuzt er den Ereignishorizont, ein Vorgang der für uns von außen prinzipiell unbeobachtbar ist. Vergleichen wir nun die scheinbare Größe (eine Winkelausdehnung am Himmel) der dunklen Zone um das Loch nach dem Einfall mit derjenigen vor dem Einfall, so stellen wir fest, dass der dunkle Bereich gewachsen ist. Der Stern “klebt nicht am Horizont”, sondern er ist in das Loch gestürzt, hat es schwerer gemacht und damit den Horizont vergrößert.

    Dieses Szenario wird von den astronomischen Beobachtungen gestützt, denn wir beobachten da draußen Kandidaten für Schwarze Löcher mit sehr unterschiedlichen Massen (wenige Sonnenmassen bis Mrd. Sonnenmassen).

    Würde Dein Szenario stimmen, Björn, so müssten wir da draußen Schwarze Löcher sehen, die alle (vor Ort) ein identisch großes, dunkles Gebiet haben und deren Umgebung von aufgesammelter, aber noch sichtbarer Materie angefüllt ist. So ist es aber nicht.

    @Stefan
    Ja, diese Bemerkung ist mir bei den CHRONOlogs auch unangenehm aufgefallen. Bei den Formulierungen muss man sehr aufpassen! Klar, rotierende Systeme in der SRT sind keine Inertialsysteme, aber dennoch gibt es Rotation in der SRT. Und in der ART: Natürlich beschreibt die Kerr-Lösung ein rotierendes System (BTW gibt es auch ein Gödel-Universum, ein rotierender Kosmos, der mit der ART beschrieben wird. Nur widerspricht der Gödel-Kosmos dem kosmologischen Prinzip, weil er nicht isotrop ist.).

  102. #102 Bjoern
    17. Januar 2010

    @Andreas:

    Die unendliche lange Dauer bezieht sich auf eine Messung, die versucht, etwas bis zum Horizont zu verfolgen.

    Also ist es so, wie ich im Kommentar am 14.1., 14:11 Uhr schrieb? Zitat:

    Oder sieht es für den Beobachter nur so aus, als ob das Verschlucken unendlich lange dauern würde, in Wirklichkeit geht es aber in endlicher Zeit vor sich?

  103. #103 Bjoern
    17. Januar 2010

    @Andreas: Noch einmal direkter: Wie passen deine beiden Aussagen: (14.1., 22:25 Uhr)

    Für einen Außenbeobachter dauert es unendlich lange, bis ein Testteilchen den Ereignishorizont erreicht,…

    und (17.1.,18:35 Uhr)

    Der Stern “klebt nicht am Horizont”, sondern er ist in das Loch gestürzt, hat es schwerer gemacht und damit den Horizont vergrößert.

    zusammen? Dauert es denn nun für den Beobachter unendlich lange, bis ein Testteilchen/Stern in das Loch stürzt, oder nicht? Wenn es unendlich lange dauert, wie kannst du dann andererseits von “ist…gestürzt” reden, also Vergangenheitsform?

    Meintest du im ersteren Zitat nicht, dass der Sturz für einen Außenbeobachter unendlich lange dauert, sondern nur, dass rein *optische* Beobachtungen des Testteilchens / des Sterns den Anschein erwecken, der Sturz *würde* unendlich lange dauern – aber trotzdem ist schon nach endlicher Zeit ein Anwachsen des SL zu messen/beobachten?

    Mit anderen Worten, ich müsste unendlich lange warten um direkt zu sehen, wie das Testteilchen / der Stern ins SL fällt, aber trotzdem sehe ich schon nach endlicher Zeit, dass die “schwarze Region” größer geworden ist?

  104. #104 david
    28. Januar 2011

    ich denke mir, es ist genauso paradox, wie das beispiel mit dem ultraschnellen zug, der vorbeifährt – der beobachter im zug, der auf eine uhr außerhalb des zuges schaut, sieht die zeit dort langsamer vergehen, als die auf seiner eigenen uhr; der beobachter außerhalb des zuges, sieht die uhr im zug aber auch langsamer ticken, als seine. beides lässt (ließe) sich aber beobachten.

  105. #105 Rica
    11. Februar 2011

    Ich glaube, dass das Bild des “Staubsaugers” (übrigens sehr charmantes Bild am Seitenanfang) einfach durch die Medien entsteht. Man denke nur an Filme wie “Das schwarze Loch”, aber auch Laien sind nicht blöd. Hawking hat sich doch auch ausführlich mit schwarzen Löchern beschäftigt und seine Bücher wurden millionenfach verkauft. Ich glaube, dass es einfach eine absolute Faszination für den Weltuntergang gibt und sich die meisten Menschen bei der Vorstellung von einem schwarzen Loch eingesaugt zu werden, einfach ein wenig gruseln können, genau wie bei spannenden Filmen oder Alien-Videos. Ob das dann unbedingt der feste Glaube ist, bezweifle ich.

  106. #106 Wolfgang Graßmann
    23. April 2011

    Das Schwarze Loch wird zum Staubsauger, wenn die Distanz des Beobachters zu ihm in die Nähe des Gravitationsradius kommt.
    Die letzte stabile Kreisbahn wird bei r=3*RG erreichte (Misner,Thorne Wheeler Page 674)
    danach gehts in den Staubsauger.
    Nun, auch ein echter Staubsauger saugstaubt nicht überall! 😉

  107. #107 SchwarzesLoch.x3
    17. Juni 2011

    Wenn man in ein Schwarzes Loch fällt, wird man immer kleiner bzw. zerquetscht. Wenn man zB. im Universum “Licht” zB. Taschenlampe hat und sie anmacht, wird das angezogen und das fällt so rein. Ein Schwarzes Loch ist sehr sehr groß. ! Bitte guckt dochmal im Wikipedia da ist ein Bild von 600km entfernung. Dankeschön das ihr das gelesen habt. 🙂

    MfG SchwarzesLoch.x3

  108. #108 Bjoern
    17. Juni 2011

    @SchwarzesLoch.x3:

    Wenn man in ein Schwarzes Loch fällt, wird man immer kleiner bzw. zerquetscht.

    Nein, man wird nicht “immer kleiner”, sondern man wird in die Länge gezogen (durch die Gezeitenkräfte; “Fach”ausdruck dafür: “Spaghettification”).

    Wenn man zB. im Universum “Licht” zB. Taschenlampe hat und sie anmacht, wird das angezogen und das fällt so rein.

    Ob das Licht “reinfällt”, hängt ganz davon ab, in welche Richtung du mit der Taschenlampe leuchtest. In den aller-, allermeisten Fällen wird das Licht in “reinfallen”, sondern nur drum herum gebeugt werden.

    Ein Schwarzes Loch ist sehr sehr groß. Bitte guckt dochmal im Wikipedia da ist ein Bild von 600km entfernung.

    Es gibt sehr sehr große Schwarze Löcher (z. B. die in den Zentren von Galaxien), aber die allermeisten Schwarzen Löcher sind eher klein. Im Wikipedia-Artikel ist ein (fiktives!) Schwarzes Loch mit 10 Sonnenmassen dargestellt. Man kann sich leicht ausrechnen, dass das einen Durchmesser von etwa 60 Kilometern haben müsste. Findest du das “sehr sehr groß”?

    (Die schwarze Scheibe, die man in dem Bild sieht, ist aber nicht unbedingt das Schwarze Loch selbst. Wie gesagt: Licht wird durch Schwarze Löcher abgelenkt, und dadurch kommt es zu Effekten ähnlich wie bei einer Linse. Man sieht ja in dem Bild schön, dass das Licht um das Schwarze Loch drum herum verzerrt wird!)

  109. #109 Alderamin
    17. Juni 2011

    @Bjoern

    Als Ergänzung könnte man noch anfügen: Schwarze Löcher sind viel weniger gefährlich, als die Sterne, aus denen sie entstanden sind. Ein Stern von 10 Sonnenmassen hat einen Durchmesser von ungefähr 5 Sonnendurchmessern, das wäre ein Radius von 3,5 Millionen km. Alles, was dem Zentrum des Sterns näher kommt, als 3,5 Millionen km, verbrennt sofort (vermutlich würde auch keine Raumsonde die zehnfache Entfernung überstehen). Dagegen kann man sich dem Zentrum des schwarzen Lochs bis auf fast 30 km nähern. Wer ist da also gruseliger?

  110. #110 Bjoern
    17. Juni 2011

    @Aldemarin: Guter Punkt. 🙂

  111. #111 kloklon
    7. September 2011

    Ich weiß es ist schon ziemlich lange her, dass hier wer was geschrieben hat, aber vielleicht kann jemand meine Frage zum Artikel beantworten:

    >> Es müsste also warten, bis ihm zufällig mal ein verirrtes Elektron nahe kommt und mit ihm kollidiert. Dann hätte es eine geringfügig größere Masse – und das Spiel geht von vorne los. >>

    Könnte man dieses Mikro-Loch denn, mal rein theoretisch, nicht “füttern” indem man es mit Teilchen bestrahlt, also aktiv und absichtlich, damit es mehr Masse bekommt? Das wäre am Anfang bestimmt schwer aber wenn man das lang genug macht bis es ein besseres (größeres) Ziel abgibt, bis es dann auf immer größere “Nahrung” “umsteigen” könnte (Protonen/Neutronen, Atome, Moleküle usw…) könnte man dann nicht ein Schwarzes Loch “züchten”?

  112. #112 Wurgl
    8. September 2011

    Nur mal so als Vergleich.

    https://de.wikipedia.org/wiki/Schwarzschildradius#Schwarzschild-Radius_und_Gravitationsradius

    Also bei 1 kg Masse hat ein Schwarzes Loch 1.4 10E-27 Meter Radius. Das ist sauklein.

    Und jetzt hier:
    https://de.wikipedia.org/wiki/Atomdurchmesser
    „Atomradien liegen in der Dimension um 10E−10 m…“

    Für 1 kg Masse brauchst du verflixt viele Atome. Und dann wäre das Schwarze Loch immer noch um 17 Größenordnungen kleiner als ein Atom.

    Deine “Züchtung” wäre also nicht sehr erfolgversprechend. Erleben würdest du ein merkbares Loch nicht, dazu wird die Spezies Mensch nicht alt genug.

  113. #113 Rico
    16. September 2011

    Schöner Text, Herr Freistetter (oder Florian?!)…
    Das hat alle meine übrigen Fragen beantwortet. Auf Galileo von Prosieben kam vorher ein ziemlich hässlicher Beitrag über Schwarze Löcher, der genau den Standpunkt vertrat, dass Schwarze Löcher “Staubsauger” sind. Das ganze wurde noch unterstützt durch Sätze wie “sie saugen alles ein, was ihnen in den weg kommt”! Ahh!!!
    Tja, die Volksverdummung schreitet voran…

  114. #114 Stephan
    12. Oktober 2011

    Sehr interessant. Hier lernt man noch mehr als bei Wikipedia 😉

    @Rico
    Galileo hat sich meines Wissens nach schon öfters geirrt. Besonders wenn es um Produkttests ging, hat die Sendung nicht ausreichend getestet und das entsprechende Produkt einfach negativ bewertet.

  115. #115 Dirk
    3. November 2011

    Anhand der Beschreibung ist ein „Schwarzes Loch“ also eigentlich gar kein „Loch“, wie im allgemeinen Sprachgebrauch verwendet wird?

  116. #116 Bullet
    3. November 2011

    Das “Loch” bezieht sich eigentlich nur darauf, daß man was reinwerfen kann, aber nichts rauskommt. Da enden aber die Gemeinsamkeiten auch schon. Das wäre auch nicht das erste Mal, daß “allgemeiner Sprachgebrauch” irreführend ist.

  117. #117 Alderamin
    3. November 2011

    @bullet

    Das wäre auch nicht das erste Mal, daß “allgemeiner Sprachgebrauch” irreführend ist.

    Stimmt, Urknall ist auch so ein Fall.

  118. #118 Hives
    20. Februar 2012

    Achsoooo ist das ^.^
    Ich muss zugeben ich hab mir das auch immer als Staubsauger vorgestellt und mit dem “Ereignishorizont” hab ich nichts anfangen können. Jetzt hab ich’s aber kapiert. Super Artikel 🙂

  119. #119 Max
    2. Januar 2013

    Das ist doch quatsch. Erstmal Leuten schwarze Löcher durch die Gase extrem, meistens Millionen fach stärker als Sonnen, so, und dann wachsen schwarze Löcher durch die einströmende Materie noch weiter und weiter. Dadurch nimmt die Gravitation zu und natürlich würden Dinge angezogen werden. Sie oder der Typ der dieses wissenschaftlich falsche Pseudowerk geschaffen haben sind einfach inkompetent.

  120. #120 Max
    2. Januar 2013

    *korrektur: Das ist doch quatsch. Erstmal Leuchten schwarze Löcher durch die Gase extrem, meistens Millionen fach stärker als Sonnen, so, und dann wachsen schwarze Löcher durch die einströmende Materie noch weiter und weiter. Dadurch nimmt die Gravitation zu und natürlich würden Dinge angezogen werden. Sie oder der Typ der dieses wissenschaftlich falsche Pseudowerk geschaffen haben sind einfach inkompetent.

  121. #121 celsus
    2. Januar 2013

    Wäre natürlich toll wenn du uns Unwissenden mal so ein leuchtendes schwarzes Loch zeigen könntest. Name der betreffenden Galaxie würde schon genügen.

  122. #122 Kallewirsch
    2. Januar 2013

    @Max

    Erstmal Leuchten schwarze Löcher durch die Gase extrem

    Du meinst die Gase um das schwarze Loch herum, falls sie dort sind?
    Das sind dann aber die Gase, die Strahlung abgeben und nicht das schwarze Loch.

    dann wachsen schwarze Löcher durch die einströmende Materie noch weiter und weiter

    Ja, und?
    Hat wer was anderes behauptet?

    Dadurch nimmt die Gravitation zu und natürlich würden Dinge angezogen werden.

    Ja, und?
    Hat wer was anderes behauptet?

    … sind einfach inkompetent.

    Und das von jemandem, dessen Lesekompetenz nicht ausreicht, einen eigentlich sehr einfach geschriebenen Artikel zu verstehen.

  123. #123 Alderamin
    2. Januar 2013

    @Max

    Natürlich haben Schwarze Löcher eine hohe Gravitation, insbesondere direkt am Ereignishorizont, da erreicht die Fluchtgeschwindigkeit die Lichtgeschwindigkeit, deswegen kann ihnen dort nichts mehr entkommen. Und Materie, die in einer Scheibe um das Schwarze Loch kreist, kann durch Reibung Bewegungsenergie in Licht und Wärme umwandeln, deshalb können sie so hell leuchten. Deswegen fällt aber trotzdem nicht alles und in jeder Entfernung auf sie zu.

    Wie oben im Text erwähnt, wenn man die Sonne durch ein Schwarzes Loch der gleichen Masse ersetzen würde, dann würde die Erde auf ihrer alten Bahn weiter kreisen, weil die Schwerkraft in dieser Entfernung vom Schwarzen Loch die selbe wäre wie vorher. Das gilt auch für die inneren Planeten. Etwas ändern würde sich nur an Orten, die derzeit innerhalb der Sonne liegen. Im Inneren der Sonne nimmt die Schwerkraft nach innen hin nämlich ab, bis zum Zentrum, wo sie 0 wird.

    Das Schwarze Loch von Sonnenmasse hätte aber nur einen Radius von 3 km, also viel weniger als die Sonne mit ihren fast 700000 km Radius, und in diesem Bereich nähme die Schwerkraft des Schwarzen Lochs weiter zu, bis auf das vieltausendfache der Sonnenschwerkraft, weil man der konzentrierten Masse so nahe käme. Aber selbst dort, außerhalb des Ereignishorizonts, könnte man noch um das Schwarze Loch kreisen, wenn man schnell genug wäre.

    Wenn ein Objekt aus dem Unendlichen (bzw. aus großer Entfernung) auf eine große Masse zufällt, wird es schneller (wegen des freien Falls). Dabei wird sein potenzielle Energie in Bewegungsenergie umgewandelt. Da keine Energie verloren geht (anders als in einer Staubscheibe mit Reibung), hat das Objekt stets genug Schwung, um aus der jeweiligen Entfernung wieder in die Ausgangsentfernung zurück zu fliegen (exakt das gleiche macht eine Schaukel oder ein Pendel, die aber im Lager und in der Luft geringe Reibungsverluste erleiden). Deswegen würde ein Objekt, das auf eine Masse (Schwarzes Loch, Stern, Planet oder was auch immer) zufällt, ihm auch wieder entkommen, wenn es nicht mit der Masse kollidierte. Eine solche Kollision ist aber nur möglich, wenn das Objekt vorher keinerlei seitliche Bewegung ausgeführt hat, sondern quasi aus dem Stillstand senkrecht auf die Masse zufällt. Ein Schwarzes Loch ist dabei ein sehr kleines Ziel – was sind 6 km Durchmesser gegen die 1,39 Millionen km des Sonnendurchmessers? Und auch die Sonne ist im großen, leeren Raum und aus den Entfernungen, die zwischen den Sternen bestehen, ein winziges Ziel. Die Chance, mit einem Schwarzen Loch (oder der es umgebenden Staubscheibe) zu kollidieren, sind also winzig und ungleich kleiner, als die Kollision mit einem sehr viel größeren Stern der gleichen Masse. Viel größer ist die Chance, an der Masse vorbei zu fallen und auf einer langgestreckten Ellipse wieder zum Ausgangspunkt zurückgeschleudert zu werden.

    Deswegen sind Sterne viel bessere “Staubsauger” als Schwarze Löcher der gleichen Masse. Und auch wenn das Supermassive Schwarze Loch im Zentrum der Milchstraße vergleichsweise groß ist (so groß wie ein Riesenstern) und eine Schwerkraft von Millionen Sonnen hat, ist es doch sehr weit weg und gegen die hunderten Milliarden Sonnenmassen der Milchstraße ein Nichts. Es wird von mehreren Sternen in unmittelbarer Nähe umkreist, die seine Anwesenheit verraten, und die dennoch nicht in es hinein fallen.

    Es ist also völlig korrekt, dass Schwarze Löche keine Staubsauger sind.

  124. #124 Rabbi
    Schweiz
    2. Januar 2013

    Habe irgendwo gehört, dass noch dieses Jahr zu beobachten (mit entsprechenden Teleskopen) sei, wie eine Gaswolke vom zentralen Schwarzen Loch der Milchstrasse “aufgesogen” würde. Ist da was dran?

  125. #126 Rabbi
    2. Januar 2013

    Florian
    Genau das meinte ich!
    hatte bloss deinen Beitrag dazu noch nicht entdeckt. Ich lese ja erst seit einigen Tagen hier mit.
    Sehr informativ! Vielen Dank!

  126. #127 Stephan Goldammer
    https://plus.google.com/106110585362718948544/posts
    16. Juli 2013

    @Florian Freistetter

    “In einem schwarzen Loch ist also Masse auf einem extrem kleinen Raum konzentriert.”

    “Das, was ein schwarzes Loch so außergewöhnlich macht, ist eine andere Eigenschaft: seine Dichte!”

    “Erst die extreme Dichte – und nicht etwa eine besonders starke “saugende” Gravitationskraft – eines schwarzen Loches führt dazu, dass die Raumzeit so stark gekrümmt wird”

    Dr. Markus Pössel vom Heidelberger Haus der Astronomie sagt in seinem Vortrag “Die häufigsten Missverständnisse über Schwarze Löcher”, dass schwarze Löcher auch die Dichte von Wasser oder Luft haben können (ab Minute 12:50).

    https://www.youtube.com/watch?v=Wfn524iifYw

    Widerspricht das nicht ihrer Aussage über die Dichte von schwarzen Löchern? Oder liegt hier das Missverständnis eines Missverständnisses über ein Missverständnis vor?

  127. #128 Alderamin
    17. Juli 2013

    @Stephan Goldammer

    Dr. Markus Pössel vom Heidelberger Haus der Astronomie sagt in seinem Vortrag “Die häufigsten Missverständnisse über Schwarze Löcher”, dass schwarze Löcher auch die Dichte von Wasser oder Luft haben können (ab Minute 12:50).

    Das stimmt aber nur für Supermassive Schwarze Löcher und auch nur dann, wenn man sich die Masse gleichverteilt über das Volumen des Ereignishorizonts denkt. Die dürfte aber kaum so verteilt sein, sondern sehr viel (unendlich?) kompakter gepackt im Zentrum desselben liegen. Es gibt keine bekannte Kraft im Universum, die dem Gravitationskollaps eines entstehenden Schwarzen Lochs entgegenwirken könnte. Wenn also keine punktförmige (oder ggf. ringförmige) Singularität herauskommen soll, dann braucht es dazu eine uns noch nicht bekannte Physik (Stringtheorie oder dergleichen). Die Masse wird im Schwarzen Loch auf jeden Fall sehr dicht gepackt sein.

    Tatsächlich kann die Gezeitenkraft an einem Supermassiven Schwarzen Loch aber so gering sein, dass man durch den Ereignishorizont fallen könnte, ohne viel zu bemerken (so jedenfalls die gängige Lehrmeinung, siehe aber hier). Da es von dort jedoch kein Zurück und auch kein Halten mehr gibt, ereilt einen die Spaghettifizierung jedoch unvermeidlich ein paar Minuten später.

  128. #129 Stephan Goldammer
    https://twitter.com/StephGoldammer
    22. Juli 2013

    @ Florian Freistetter

    Ich habe zwar bisher keine Antwort auf die Frage mit der Dichte bekommen, bleibe aber am Ball 😉

    Ich hätte noch weitere Fragen:

    Harald Lesch hat in einem Alpha-Centauri-Video gesagt, man könne mit einem Photonensegel gegen die Sonne kreuzen. Wie das Kreuzen beim Segeln auf dem Wasser funktioniert kann ich mir noch herleiten, aber gegen die Sonne kreuzen? Der (Photonen)Druck müsste mich doch wegtreiben?

    In “Alpha Centauri – Wie kann man auf Licht segeln?” bei Minute 09:00:

    https://www.br.de/fernsehen/br-alpha/sendungen/alpha-centauri/alpha-centauri-licht-2006_x100.html

    Noch eine Frage zu einem anderen Gebiet:

    Der Begriff Wellenfunktion hat mich bisher insofern verwirrt, dass ich mit dem Begriff „Welle“ automatisch c als maximale Geschwindigkeit assoziiert habe. Da die Wellenfunktion aber mit „überlicht bzw. instantan gestaucht“ wird, passt doch der Begriff „Welle“ in “Wellenfunktion” nicht wirklich gut (bezogen auf “Geschwindigkeit” und den handelsüblichen Gebrauch als elekro-magnetische (etc.) Welle, also (Licht)Geschwindigkeits-Welle)?

    Würde mich über eine Antwort sehr freuen.

    Viele Grüße

  129. #130 Alderamin
    22. Juli 2013

    @Stephan Goldammer

    Ich habe zwar bisher keine Antwort auf die Frage mit der Dichte bekommen, bleibe aber am Ball

    Öh, nein?

  130. #131 PDP10
    23. Juli 2013

    @Stephan Goldammer:

    “überlicht bzw. instantan gestaucht“

    ?? Wo hast du denn die Formulierung her?

    Meinst du vielleicht das die Wellenfunktion bei einer Messung “kollabiert”?

    Die Sache mit der Welle kommt übrigens daher, dass die Schrödinger Gleichung (quasi die Grundgleichung der Quantenmechanik) eine Differentialgleichung ist, die eine Welle beschreibt.

    Genauer gesagt, sind die Lösungen Funktionen die Wellen beschreiben.

    Und experimentell kann man sehen, dass sich Quantensysteme (Elektronen zB) eben wie Wellen verhalten. Sie werden zB wie Wellen an einem Spalt gebeugt, statt wie Teilchen einfach durchzugehen oder nicht.

    Hilft das erstmal weiter?

  131. #132 Stephan Goldammer
    https://twitter.com/StephGoldammer
    23. Juli 2013

    @Alderamin

    Doch ich habe ihren Post gelesen, wollte aber die Antwort von Herrn Freistetter abwarten. Bin erst seit kurzem hier, habe mich durch viele sehr interessante Beiträge hindurchgelesen und bisher nur: Schwarzes Loch = Extrem hohe Dichte, gefunden. Eine Einschränkung dieser All-Aussage gab es bisher hier auf dem Blog nicht. Daher warte ich mal die Antwort ab. Als ich den Beitrag von Herrn Pössel zum ersten Mal sah, war ich sehr überrascht, da ich wie 99% aller “Laien”, bisher davon ausging, dass die Dichte eines schwarzen Lochs immer sehr hoch ist. Schwarze Löcher mit Wasser- oder gar Luft-Dichten waren bisher außerhalb meines Vorstellungsvermögens (was auch daran liegt das irgendwie “alle” sagen, die Dichte ist sehr hoch). Daher wollt ich mal ganz genau nachfragen.

    @PDP10

    Das sich Quantensysteme wie Wellen verhalten und interferieren usw., verstehe ich schon. Aber: Der “Geschwindigkeitsanteil” im Begriff “Welle” ist einerseits die Lichtgeschwindigkeit (z.B. elektro-magn. Welle), anderseits enthält der Begriff “Welle” in der quantenmechanischen Wellenfunktion keinen Geschwindigkeitsanteil, denn kollabiert wird hier in “Nullzeit”. Das verwirrt mich. Mal hat der Begriff “Welle” eine (Ausbreitungs)Geschwindigkeit(sobergrenze), mal nicht.

  132. #133 Florian Freistetter
    23. Juli 2013

    @stephan Ach so… naja, nachdem alderamin schon ne kompetente Antwort gegeben hat, dachte ich, ich muss nichts mehr sagen. Wenn alderamin was falsches gesagt hätte, hätte ich es korrigiert… ich bin aber auch gerade unterwegs und habe schlechten Empfang und kann nur sporadisch online gehen. Aber alderamin kennt sich aus, seinen antworten kannst du vertrauen.

  133. #134 PDP10
    23. Juli 2013

    @Stephan Goldammer:

    Häng nicht zu sehr am Begriff der Elektromagnetischen Welle.

    Die Wellenfunktionen in der QM beschreiben Wahrscheinlichkeitswellen.
    Und die Ausbreitungsgeschwindigkeit ist keineswegs immer c.

    Vielleicht hilft dir der Artikel über die De Broglie Wellenlänge in der Wikipedia weiter:

    https://de.wikipedia.org/wiki/De-Broglie-Wellenl%C3%A4nge#Die_De-Broglie-Wellenl.C3.A4nge

    Der ist zwar kurz, erklärt das ganze aber recht anschaulich.

    Das ganze hat erstmal mit dem Zusammenbruch der Wellenfunktion bei einer Messung nichts zu tun.

    Dieser “Zusammenbruch” erfolgt nach der reinen Lehre tatsächlich instantan.

    Mit sehr trickreichen Experimenten kann man die Wellen allerdings beim kollabieren “beobachten”, bzw. überlagerte (verschränkte) Zustände studieren, bevor sie sich für die eine oder andere Richtung entscheiden.

    Für sowas wurde übrigens letztes Jahr der Nobelpreis an Serge Haroche und David Wineland verliehen.

    Die beiden haben Experimente gemacht, bei denen man gewissermassen Schrödingers Katze vor dem Öffnen der Kiste studieren kann 🙂

  134. #135 Stephan Goldammer
    https://twitter.com/StephGoldammer
    31. Juli 2013

    Oh, ich habe eine schöne Erklärung gefunden. Die physikalische, elektromagnetische Welle und die mathematische Wahrscheinlichkeits-Welle müssen voneinander unterschieden werden.

    Dummerweise heißen beide “Welle”, das hatte mich bisher immer verwirrt.

    Wer das mal nachlesen möchte, bei Franz Embacher, Universität Wien:

    https://homepage.univie.ac.at/franz.embacher/MERLIN_MPI/konzept.htm

    Jetzt ist noch das mit dem “Gegen die Sonne kreuzen” von Harald Lesch offen. Kann mir da jemand weiterhelfen?

  135. #136 Pascal Florian
    Sachsen-Anhalt
    8. November 2013

    Ich denke, wenn ein Schwarzes Loch nicht saugt, hat es trotzdem noch die Anziehungskraft seines Normalzustandes.
    Für mich, als Laie, ist es natürlich interessant was passiert wenn man sich das Loch nähert. Es ist ja NIX, aber dennoch hat es eine Dichte (?), also müsste man wirklich irgendwie aufgelöst werden. Schwer zu erklären…

  136. #137 Florian Freistetter
    8. November 2013
  137. #138 Bullet
    8. November 2013

    @Pascal Florian:
    ein Schwarzes Loch ist NIX? Huch? wie kommst du denn darauf?

  138. #139 Alderamin
    8. November 2013

    @Pascal Florian

    Das Schwarze Loch hat die Masse des Objekts, aus dem es entstanden ist – normalerweise der Kern eines als Supernova explodierten großen Sterns; der Stern muss dafür wenigstens 10 bis 20 Sonnenmassen gehabt haben, der Kern hat dann 2,5 Sonnenmassen oder mehr.

    Die Anziehungskraft des Schwarzen Lochs entspricht der Masse, die in ihm steckt, also der von ein paar Sonnenmassen. Man kann sich einem Stern der entsprechenden Masse nicht mehr als bis zu seiner Oberfläche nähern (und auch das nur theoretisch!), die Schwerkraft kann also nicht größer werden, als an seiner Oberfläche. Innerhalb des Sterns nimmt sie zum Zentrum hin wieder ab bis auf 0 (was einem in dem tausende Grad heißen Plasma aber nicht viel nutzte).

    Ein Schwarzes Loch ist aber viel kleiner, man kann sich ihm bis zum Ereignishorizont nähern, wo nicht einmal mehr Licht entkommen kann, und sogar hindurch fallen, weil dies keine feste Oberfläche, sondern nur gekrümmter Raum ist. Der Ereignishorizont eines Schwarzen Lochs von einer Sonnenmasse hätte (wenn es so kleine Schwarze Löcher gäbe; wie gesagt braucht es 2,5-3 Sonnenmassen, wenn es in einer Supernova gebildet wurde) einen Radius von nur 3 km. Die Schwerkraft nimmt mit dem Quadrat der Entfernung ab, wenn man also halb so nahe an die Masse herankommt, ist die Schwerkraft 4-mal so hoch. Auf der Sonnenoberfläche, ca. 695000 km von ihrem Zentrum, ist die Schwerkraft 27,9-mal so hoch wie auf der Erde (1 G). In 3 km Entfernung von einem Schwarzen Loch von einer Sonnemasse wäre sie demnach 2,97 G * (695000 km / 3 km)² = 1,5 Billionen G.

    Das alleine wäre noch nicht das Problem. Schlimmer ist, dass die Schwerkraft, die an Deinen Füßen zieht, eine andere ist, als an Deinem Kopf. Über Deine Körperlänge variiert die Schwerkraft. Nehmen wir 1,8 m an, Füße nach unten, dann ist bei 3000 Meter Radius des Ereignishorizonts der Kopf 1,0003-mal weiter vom Massenzentrum weg als der Bauch, während die Füße 0,9997 mal näher dran sind. Die Schwerkraft unterscheidet sich dann um den Faktor (1,0003²-0,9997²)/(1,0003²*0,9997²) = 0,0012 oder 0,12% – von 1,5 Billionen G. Das sind 1,8 Milliarden G Unterschied. An Deinem Hals zöge Dein fast 2-Milliardenfaches Gewicht! Das würde Dich einfach in Stücke reißen.

    Interessanterweise gibt es große, sogenannte Supermassive Schwarze Löcher mit Millionen Sonnenmassen im Zentrum fast aller Galaxien, auch unserer Milchstraße. Da ist der Ereignishorizont bis zu hunderte Millionen km groß und der Unterschied der Schwerkraft zwischen Kopf- und Fußende ist nur winzig, d.h. man kann problemlos durch den Ereignishorizont fallen, ohne etwas zu merken. Dahinter gibt es jedoch keine Rückkehr mehr und die Schwerkraft nimmt innerhalb des Schwarzen Lochs weiter zu, bis einen die Differenz zwischen Kopf- und Fußende am Ende doch irgendwo zerreisst.

    Wegen Deiner hohen Geschwindigkeit (fast genau Lichtgeschwindigkeit) würde Dir das alles jedoch sehr schnell erscheinen, die Flugstrecke erschiene stark gestaucht. Es ginge alles viel schneller, als Du es bemerken könntest.

  139. #140 Alderamin
    8. November 2013

    @Pascal Florian

    2,97 G * (695000 km / 3 km)²

    Tippfehler, muss natürlich 27,9 G * (695000 km / 3 km)² lauten, aber das Ergebnis war korrekt, 1,5 Billionen G.

  140. #141 Dart1976
    9. Mai 2015

    So und wie kommt die Gravitation aus dem Schwarzenloch?

    Da liegt der Punkt wo ich mit Einstein ein Problem habe, besonders bei der Frage, ob es Raum und Zeit ohne Materie geben kann.
    Übrigens auch mit Hawking!
    No-Hair-Theorem aber man hat die Information über seine Masse und das ohne Zeitliche Singularität, in Echtzeit! Verschluckte Masse „muss“ sich immer noch auf die Umgebung Gravitativ auswirken! Würde sonst sicher „tierisch“ auffallen.

    Übrigens auch die Higgs Bosonen halte ich für einen Phsikalischen Witz, oder sind die etwa über Lichtschnell?

    Einzige Erklärung die mir so einfallt, ist das Masse nicht die Raumzeit krümmt, sonndern die Ursache der Raumzeit ist. Und somit nicht E=mc² sondern m=E/c² also das was wir als Masse Beschreiben ist nicht weiter als der Verhältniss von Raum und Zeit und die jeweilige Energie Dichte. Wobei Energie ohne Raum und Zeit und Masse nicht existiert, siehe die Einheit zb.: kg·m2·s−2

  141. #142 PDP10
    9. Mai 2015

    @Dart1976:

    “So und wie kommt die Gravitation aus dem Schwarzenloch?”

    Darüber hat Florian früher schon mal was geschrieben:

    https://scienceblogs.de/astrodicticum-simplex/2014/08/04/wie-kommt-die-gravitation-aus-dem-schwarzen-loch-heraus/

    “Und somit nicht E=mc² sondern m=E/c²”

    Herzlichen Glückwunsch. Du hast die Definition der Ruhemasse hingeschrieben.

    Das hat allerdings mit der ART nicht viel zu tun.
    Die Dinge sind da ein wenig komplizierter …

  142. #143 Dart1976
    9. Mai 2015

    Danke für die Antwort und den Link, jedoch naja…

    Natürlich muss die Gravitationskraft nicht aus den Loch.
    Und die Ausbreitungsgeschwindigkeit der Gravitation ist auch eher Nebensache, da sie sich einfach mit der Maximalen Massenänderungsgeschwindigkeit ausbreitet, also genau so schnell wie die Masse weg ist, ist auch die Gravitation weg, und somit gilt mal sicher maximal Lichtgeschwindigkeit.

    Und natürlich ist die Formel grob vereinfacht und laienhaft, aber ich meine ja die Idee dahinter.
    Materie ist ja eine Welle, und somit ist die Raumzeit genau diese Welle und deren Überlagerungen der Materie-wellen.
    Die Frage dich mich beschäftige, Gravitation soll ja unendliche Reichweite habe, was ich Einstein nicht abkaufe.
    Wie groß wäre das Universum wenn es nur genau die Mindest- Masse von 1 Plancksche Einheit hätte da müsste doch genau 1 Ls raus kommen.

  143. #144 PDP10
    9. Mai 2015

    @Dart1976:

    “Und die Ausbreitungsgeschwindigkeit der Gravitation ist auch eher Nebensache, da sie sich einfach mit der Maximalen Massenänderungsgeschwindigkeit ausbreitet, also genau so schnell wie die Masse weg ist, ist auch die Gravitation weg, und somit gilt mal sicher maximal Lichtgeschwindigkeit.”

    Ich verstehe nicht, was das bedeutet.

    “Materie ist ja eine Welle, und somit ist die Raumzeit genau diese Welle und deren Überlagerungen der Materie-wellen.”

    Ich verstehe hier auch nicht, was das bedeutet.

  144. @Dart1976:“So und wie kommt die Gravitation aus dem Schwarzenloch?”

    Siehe hier: https://scienceblogs.de/astrodicticum-simplex/2014/08/04/wie-kommt-die-gravitation-aus-dem-schwarzen-loch-heraus/

  145. #146 gangstar222
    7. Juni 2015

    Danke vielmals Florian ich bin 12 und habe das thema Universu als projekt. Auf dieser Seite lerne ich sehr viel.
    Vielen dank.